SlideShare a Scribd company logo
1 of 64
Download to read offline
TRƯỜNG ĐẠI HỌC BÁCH KHOA HÀ NỘI
VIỆN TOÁN ỨNG DỤNG & TIN HỌC
BÙI XUÂN DIỆU
Bài Giảng
OLYMPIC SINH VIÊN MÔN ĐẠI SỐ
ĐỊNH THỨC, HỆ PHƯƠNG TRÌNH TUYẾN TÍNH, MA TRẬN VÀ ÁNH XẠ
TUYẾN TÍNH, ĐA THỨC
Tóm tắt lý thuyết, các ví dụ, bài tập và lời giải
Dresden (Germany) - 2012
MỤC LỤC
Mục lục. . . . . . . . . . . . . . . . . . . . . . . . . . . . . . . 1
Chương 1 . Ma trận - Định thức . . . . . . . . . . . . . . . . . . . . . . . 5
1 Định thức . . . . . . . . . . . . . . . . . . . . . . . . . . . . . . . . . . . . . . . 5
1.1 Các tính chất cơ bản của định thức . . . . . . . . . . . . . . . . . . . . 5
1.2 Các định thức đặc biệt . . . . . . . . . . . . . . . . . . . . . . . . . . . 6
1.3 Bài tập . . . . . . . . . . . . . . . . . . . . . . . . . . . . . . . . . . . . 11
2 Định thức con và phần phụ đại số . . . . . . . . . . . . . . . . . . . . . . . . . 13
2.1 Các định nghĩa và tính chất . . . . . . . . . . . . . . . . . . . . . . . . 13
2.2 Bài tập . . . . . . . . . . . . . . . . . . . . . . . . . . . . . . . . . . . . 14
3 Phần bù Schur . . . . . . . . . . . . . . . . . . . . . . . . . . . . . . . . . . . . 16
3.1 Các định nghĩa và tính chất . . . . . . . . . . . . . . . . . . . . . . . . 16
3.2 Bài tập . . . . . . . . . . . . . . . . . . . . . . . . . . . . . . . . . . . . 17
Chương 2 . Không gian véctơ - Ánh xạ tuyến tính . . . . . . . . . . . . . . 19
1 Không gian đối ngẫu - Phần bù trực giao . . . . . . . . . . . . . . . . . . . . . 19
1.1 Không gian đối ngẫu . . . . . . . . . . . . . . . . . . . . . . . . . . . . 19
1.2 Phần bù trực giao . . . . . . . . . . . . . . . . . . . . . . . . . . . . . . 21
1.3 Bài tập . . . . . . . . . . . . . . . . . . . . . . . . . . . . . . . . . . . . 21
2 Hạt nhân và ảnh - Không gian thương . . . . . . . . . . . . . . . . . . . . . . 22
2.1 Hạt nhân và ảnh . . . . . . . . . . . . . . . . . . . . . . . . . . . . . . 22
2.2 Không gian thương . . . . . . . . . . . . . . . . . . . . . . . . . . . . . 23
2.3 Bài tập . . . . . . . . . . . . . . . . . . . . . . . . . . . . . . . . . . . . 24
3 Cơ sở của không gian véctơ - Độc lập tuyến tính . . . . . . . . . . . . . . . . . 25
3.1 Bài toán đổi cơ sở . . . . . . . . . . . . . . . . . . . . . . . . . . . . . . 25
3.2 Bài tập . . . . . . . . . . . . . . . . . . . . . . . . . . . . . . . . . . . . 25
4 Hạng của ma trận . . . . . . . . . . . . . . . . . . . . . . . . . . . . . . . . . . 27
4.1 Các tính chất của hạng của ma trận . . . . . . . . . . . . . . . . . . . 27
4.2 Bài tập . . . . . . . . . . . . . . . . . . . . . . . . . . . . . . . . . . . . 28
1
2 MỤC LỤC
Chương 3 . Dạng chính tắc của ma trận và toán tử tuyến tính . . . . . . . . 31
1 Vết của ma trận . . . . . . . . . . . . . . . . . . . . . . . . . . . . . . . . . . . 31
2 Cấu trúc của tự đồng cấu . . . . . . . . . . . . . . . . . . . . . . . . . . . . . . 32
2.1 Trị riêng và véctơ riêng . . . . . . . . . . . . . . . . . . . . . . . . . . . 32
2.2 Tự đồng cấu chéo hoá được . . . . . . . . . . . . . . . . . . . . . . . . . 33
2.3 Đa thức tối tiểu . . . . . . . . . . . . . . . . . . . . . . . . . . . . . . . 34
2.4 Bài tập . . . . . . . . . . . . . . . . . . . . . . . . . . . . . . . . . . . . 35
3 Dạng chuẩn của ma trận . . . . . . . . . . . . . . . . . . . . . . . . . . . . . . 40
3.1 Dạng chuẩn Jordan của ma trận . . . . . . . . . . . . . . . . . . . . . 40
3.2 Dạng chuẩn Frobenius . . . . . . . . . . . . . . . . . . . . . . . . . . . 41
3.3 Bài tập . . . . . . . . . . . . . . . . . . . . . . . . . . . . . . . . . . . . 42
4 Biểu diễn ma trận . . . . . . . . . . . . . . . . . . . . . . . . . . . . . . . . . . 43
4.1 Rút gọn một ma trận về ma trận dạng đường chéo đơn giản . . . . . . 43
4.2 Biểu ma trận dưới dạng tọa độ cực . . . . . . . . . . . . . . . . . . . . 43
4.3 Biểu diễn Schur . . . . . . . . . . . . . . . . . . . . . . . . . . . . . . . 44
4.4 Biểu diễn Lanczos . . . . . . . . . . . . . . . . . . . . . . . . . . . . . . 44
4.5 Bài tập . . . . . . . . . . . . . . . . . . . . . . . . . . . . . . . . . . . . 44
Chương 4 . Các ma trận có dạng đặc biệt . . . . . . . . . . . . . . . . . . 45
1 Ma trận đối xứng - Ma trận Hermitian . . . . . . . . . . . . . . . . . . . . . . 45
1.1 Các định nghĩa và tính chất . . . . . . . . . . . . . . . . . . . . . . . . 45
1.2 Bài tập . . . . . . . . . . . . . . . . . . . . . . . . . . . . . . . . . . . . 46
2 Ma trận phản xứng . . . . . . . . . . . . . . . . . . . . . . . . . . . . . . . . . 47
2.1 Các định nghĩa và tính chất . . . . . . . . . . . . . . . . . . . . . . . . 47
2.2 Bài tập . . . . . . . . . . . . . . . . . . . . . . . . . . . . . . . . . . . . 47
3 Ma trận trực giao - Phép biến đổi Cayley . . . . . . . . . . . . . . . . . . . . . 48
3.1 Các định nghĩa và tính chất . . . . . . . . . . . . . . . . . . . . . . . . 48
3.2 Bài tập . . . . . . . . . . . . . . . . . . . . . . . . . . . . . . . . . . . . 48
4 Ma trận chuẩn tắc . . . . . . . . . . . . . . . . . . . . . . . . . . . . . . . . . . 50
4.1 Các định nghĩa và tính chất . . . . . . . . . . . . . . . . . . . . . . . . 50
4.2 Bài tập . . . . . . . . . . . . . . . . . . . . . . . . . . . . . . . . . . . . 50
5 Ma trận luỹ linh . . . . . . . . . . . . . . . . . . . . . . . . . . . . . . . . . . . 52
5.1 Các định nghĩa và tính chất . . . . . . . . . . . . . . . . . . . . . . . . 52
5.2 Bài tập . . . . . . . . . . . . . . . . . . . . . . . . . . . . . . . . . . . . 52
6 Toán tử chiếu - Ma trận lũy đẳng . . . . . . . . . . . . . . . . . . . . . . . . . 54
6.1 Các định nghĩa và tính chất . . . . . . . . . . . . . . . . . . . . . . . . 54
6.2 Bài tập . . . . . . . . . . . . . . . . . . . . . . . . . . . . . . . . . . . . 54
7 Ma trận đối hợp . . . . . . . . . . . . . . . . . . . . . . . . . . . . . . . . . . . 57
MỤC LỤC 3
8 Ma trận hoán vị (hay còn gọi là ma trận giao hoán) . . . . . . . . . . . . . . . 58
8.1 Định nghĩa . . . . . . . . . . . . . . . . . . . . . . . . . . . . . . . . . . 58
8.2 Bài tập . . . . . . . . . . . . . . . . . . . . . . . . . . . . . . . . . . . . 58
Chương 5 . Các bất đẳng thức ma trận. . . . . . . . . . . . . . . . . . . . 59
1 Các bất đẳng thức cho ma trận đối xứng và Hermitian . . . . . . . . . . . . . 59
1.1 Các định lý cơ bản . . . . . . . . . . . . . . . . . . . . . . . . . . . . . . 59
1.2 Bài tập . . . . . . . . . . . . . . . . . . . . . . . . . . . . . . . . . . . . 60
2 Các bất đẳng thức cho trị riêng . . . . . . . . . . . . . . . . . . . . . . . . . . 61
2.1 Các bất đẳng thức cơ bản . . . . . . . . . . . . . . . . . . . . . . . . . . 61
2.2 Bài tập . . . . . . . . . . . . . . . . . . . . . . . . . . . . . . . . . . . . 62
Chương 6 . Đa thức . . . . . . . . . . . . . . . . . . . . . . . . . . . . . 63
4 MỤC LỤC
CHƯƠNG 1
MA TRẬN - ĐỊNH THỨC
§1. ĐỊNH THỨC
1.1 Các tính chất cơ bản của định thức
Định thức của một ma trận vuông A = (aij)n
1 cấp n là tổng luân phiên
∑
σ
(−1)σ
a1σ(1)a2σ(2) . . . anσ(n),
ở đó tổng trên được lấy qua tất cả các phép hoán vị σ ∈ Sn. Định thức của ma trận A được
kí hiệu là det A hoặc |A|, nếu det A 6= 0 ta nói A là ma trận khả nghịch (không suy biến).
Các tính chất sau đây thường được sử dụng để tính định thức của một ma trận. Các bạn
có thể kiểm chứng hoặc chứng minh chúng một cách dễ dàng.
1. Nếu đổi chỗ hai hàng (hoặc hai cột) của ma trận A thì định thức của nó đổi dấu. Nói
riêng, nếu ma trận A có hai hàng (cột)giống nhau thì det A = 0.
2. Nếu A, B và C là các ma trận vuông cùng cấp thì det
A C
0 B
!
= det A. det B.
3. det A =
n
∑
j=1
(−1)i+jMi,j, ở đó Mij là định thức của ma trận thu được từ A bằng cách bỏ
đi hàng thứ i và cột thứ j của nó. Công thức này còn được gọi là công thức khai triển
định thức theo hàng. Các bạn có thể tự viết công thức khai triển định thức theo cột
một cách tương tự.
4.
λ1α1 + µ1β1 a12 . . . a1n
.
.
.
.
.
. . . .
.
.
.
λnαn + µnβn an2 . . . ann
= λ
α1 a12 . . . a1n
.
.
.
.
.
. . . .
.
.
.
αn an2 . . . ann
+ µ
β1 a12 . . . a1n
.
.
.
.
.
. . . .
.
.
.
βn an2 . . . ann
5
6 Chương 1. Ma trận - Định thức
5. det(AB) = det A det B.
6. det(AT) = detA.
1.2 Các định thức đặc biệt
Định thức Vandermonde
Ma trận Vandermonde cấp n là ma trận vuông cấp n có dạng
Vn(a1, a2, ..., an) =








1 1 . . . 1 1
a1 a2 . . . an−1 an
a2
1 a2
2 . . . a2
n−1 a2
n
.
.
.
.
.
.
...
.
.
.
.
.
.
an−1
1 an−1
2 . . . an−1
n−1 an−1
n








Định lý 1.1. Chứng minh rằng det Vn(a1, a2, ..., an) = ∏
16i<j6n
(aj − ai). Từ đó suy ra hệ
Vn(a1, a2, ..., an).X = 0 chỉ có nghiệm tầm thường khi và chỉ khi a1, a2, . . . , an đôi một phân
biệt.
Một ứng dụng thú vị của định thức Vandermonde là bài toán sau:
Bài tập 1.1. Cho A là một ma trận vuông cấp n. Khi đó
An
= 0 ⇔ tr(Ak
) = 0, k = 0, 1, 2, . . . , n
Chứng minh. ⇒ Nếu An = 0 thì A là một ma trận lũy linh, do đó A chỉ có các trị riêng
bằng 0, nên Ak cũng chỉ có các trị riêng bằng 0 với mọi k. Suy ra điều phải chứng minh.
⇐ Giả sử các giá trị riêng của A là λ1, λ2, . . . , λn. Khi đó từ tr(Ak) = 0, k = 0, 1, 2, . . . , n ta
có hệ phương trình: 














λ1 + λ2 + . . . + λn = 0
λ2
1 + λ2
2 + . . . + λ2
n = 0
.
.
.
λn
1 + λn
2 + . . . + λn
n = 0
(1.1)
hay
Vn(λ1, λ2, . . . , λn)(λ1, λ2, . . . , λn)T
= 0.
Ta sẽ chứng minh tất cả các giá trị riêng của A bằng nhau. Thật vậy:
Nếu λi đôi một phân biệt thì định thức Vandermonde khác không, hệ phương trình trên
chỉ có nghiệm duy nhất λ1, λ2, . . . , λn = 0. Mâu thuẫn.
1. Định thức 7
Ngược lại, không mất tính tổng quát, giả sử λ1 = λ2 và không một giá trị λi còn lại nào
bằng nhau. Khi đó hệ phương trình được viết lại dưới dạng
Vn−1(λ2, . . . , λn)(2λ2, . . . , λn)T
= 0
Lập luận tương tự ta có λ2 = . . . = λn = 0, mâu thuẫn.
Vậy tất cả các trị riêng của A bằng nhau và do đó bằng 0.
Bài tập 1.2. Chứng minh rằng với các số nguyên k1 < k2 < ... < kn bất kì thì
det Vn(k1, k2, ..., kn)
det Vn(1, 2, ..., n)
là một số nguyên.
Bài tập 1.3. Cho W là ma trận có được từ ma trận V = Vn(a1, a2, ..., an) bằng cách thay
hàng (an−1
1 , an−1
2 , ..., an−1
n ) bởi hàng (an
1 , an
2 , ..., an
n). Chứng minh rằng
det W = (a1 + a2 + ... + an) det V.
Bài tập 1.4. Chứng minh rằng
det









1 1 . . . 1 1
a1 a2 . . . an−1 an
.
.
.
.
.
.
...
.
.
.
.
.
.
an−2
1 an−2
2
.
.
. an−2
n−1 an−2
n
a2a3...an a1a3...an . . . a1a2...an−2an a1a2...an−1









= (−1)n−1
. det Vn(a1, a2, ..., an)
Chứng minh. • Nếu a1, a2, . . . , an 6= 0 thì nhân cột thứ nhất với a1, cột thứ hai với a2, . . . ,
cột thứ n với an rồi chia cho a1a2 . . . an ta được
det









1 1 . . . 1 1
a1 a2 . . . an−1 an
.
.
.
.
.
.
...
.
.
.
.
.
.
an−2
1 an−2
2
.
.
. an−2
n−1 an−2
n
a2a3...an a1a3...an . . . a1a2...an−2an a1a2...an−1









=
1
a1a2 . . . an
. det









a1 a2 . . . an−1 an
a2
1 a2
2 . . . a2
n−1 a2
n
.
.
.
.
.
.
...
.
.
.
.
.
.
an−1
1 an−1
2
.
.
. an−1
n−1 an−1
n
1 1 . . . 1 1









= (−1)n−1. det Vn(a1, a2, ..., an)
• Trường hợp có ít nhất một trong các số a1, a2, . . . , an bằng 0 (xét riêng).
8 Chương 1. Ma trận - Định thức
Bài tập 1.5. Cho f1(x), f2(x), ..., fn(x) là các đa thức bậc không quá n − 2. Chứng minh
rằng với mọi số a1, a2, . . . , an ta có
f1(a1) f1(a2) . . . f1(an)
f2(a1) f2(a2) . . . f2(an)
.
.
.
.
.
.
...
.
.
.
fn(a1) fn(a2) . . . fn(an)

More Related Content

What's hot

đề Cương ôn tập môn pháp luật đại cương haui
đề Cương ôn tập môn pháp luật đại cương hauiđề Cương ôn tập môn pháp luật đại cương haui
đề Cương ôn tập môn pháp luật đại cương haui
Huynh ICT
 
Chương 2 phép duy vật biện chứng
Chương 2 phép duy vật biện chứngChương 2 phép duy vật biện chứng
Chương 2 phép duy vật biện chứng
Su Chann
 
Đề thi trắc nghiệm Xác suất thống kê có lời giải
Đề thi trắc nghiệm Xác suất thống kê có lời giảiĐề thi trắc nghiệm Xác suất thống kê có lời giải
Đề thi trắc nghiệm Xác suất thống kê có lời giải
希夢 坂井
 
Thời kỳ quá độ lên CHủ nghĩa Xã hội
Thời kỳ quá độ lên CHủ nghĩa Xã hộiThời kỳ quá độ lên CHủ nghĩa Xã hội
Thời kỳ quá độ lên CHủ nghĩa Xã hội
Bích Phương
 
De xstk k11
De xstk k11De xstk k11
De xstk k11
dethinhh
 
Giao trinh quan tri hoc dai cuong
Giao trinh quan tri hoc dai cuongGiao trinh quan tri hoc dai cuong
Giao trinh quan tri hoc dai cuong
Phi Phi
 
đề Thi xác suất thống kê và đáp án
đề Thi xác suất thống kê và đáp ánđề Thi xác suất thống kê và đáp án
đề Thi xác suất thống kê và đáp án
Học Huỳnh Bá
 
Tiểu luận tư tưởng hồ chí minh về con người mới
Tiểu luận tư tưởng hồ chí minh về con người mới Tiểu luận tư tưởng hồ chí minh về con người mới
Tiểu luận tư tưởng hồ chí minh về con người mới
Võ Thùy Linh
 

What's hot (20)

đề Cương ôn tập môn pháp luật đại cương haui
đề Cương ôn tập môn pháp luật đại cương hauiđề Cương ôn tập môn pháp luật đại cương haui
đề Cương ôn tập môn pháp luật đại cương haui
 
Chương 3 lý thuyết về hành vi của người tiêu dùng
Chương 3 lý thuyết về hành vi của người tiêu dùngChương 3 lý thuyết về hành vi của người tiêu dùng
Chương 3 lý thuyết về hành vi của người tiêu dùng
 
chuong 3. quan he
chuong 3. quan hechuong 3. quan he
chuong 3. quan he
 
Chương 2 phép duy vật biện chứng
Chương 2 phép duy vật biện chứngChương 2 phép duy vật biện chứng
Chương 2 phép duy vật biện chứng
 
Đề thi trắc nghiệm Xác suất thống kê có lời giải
Đề thi trắc nghiệm Xác suất thống kê có lời giảiĐề thi trắc nghiệm Xác suất thống kê có lời giải
Đề thi trắc nghiệm Xác suất thống kê có lời giải
 
Thời kỳ quá độ lên CHủ nghĩa Xã hội
Thời kỳ quá độ lên CHủ nghĩa Xã hộiThời kỳ quá độ lên CHủ nghĩa Xã hội
Thời kỳ quá độ lên CHủ nghĩa Xã hội
 
De xstk k11
De xstk k11De xstk k11
De xstk k11
 
Tài chính doanh nghiệp
Tài chính doanh nghiệp Tài chính doanh nghiệp
Tài chính doanh nghiệp
 
Giao trinh quan tri hoc dai cuong
Giao trinh quan tri hoc dai cuongGiao trinh quan tri hoc dai cuong
Giao trinh quan tri hoc dai cuong
 
Bo de-thi-va-loi-giai-xac-xuat-thong-ke
Bo de-thi-va-loi-giai-xac-xuat-thong-keBo de-thi-va-loi-giai-xac-xuat-thong-ke
Bo de-thi-va-loi-giai-xac-xuat-thong-ke
 
Toán cao-cấp-1
Toán cao-cấp-1Toán cao-cấp-1
Toán cao-cấp-1
 
Tiếng Anh cơ bản 2 cho các trường đại học
Tiếng Anh cơ bản 2 cho các trường đại họcTiếng Anh cơ bản 2 cho các trường đại học
Tiếng Anh cơ bản 2 cho các trường đại học
 
201-bai-tap-phuong-trinh-vi-phan
 201-bai-tap-phuong-trinh-vi-phan 201-bai-tap-phuong-trinh-vi-phan
201-bai-tap-phuong-trinh-vi-phan
 
ĐẠO ĐỨC TRONG QUAN HỆ VỚI CÁC ĐỐI TƯỢNG HỮU QUAN  TS. BÙI QUANG XUÂN
ĐẠO ĐỨC TRONG QUAN HỆ VỚI CÁC ĐỐI TƯỢNG HỮU QUAN    TS. BÙI QUANG XUÂNĐẠO ĐỨC TRONG QUAN HỆ VỚI CÁC ĐỐI TƯỢNG HỮU QUAN    TS. BÙI QUANG XUÂN
ĐẠO ĐỨC TRONG QUAN HỆ VỚI CÁC ĐỐI TƯỢNG HỮU QUAN  TS. BÙI QUANG XUÂN
 
đề Thi xác suất thống kê và đáp án
đề Thi xác suất thống kê và đáp ánđề Thi xác suất thống kê và đáp án
đề Thi xác suất thống kê và đáp án
 
Tiểu luận tư tưởng hồ chí minh về con người mới
Tiểu luận tư tưởng hồ chí minh về con người mới Tiểu luận tư tưởng hồ chí minh về con người mới
Tiểu luận tư tưởng hồ chí minh về con người mới
 
Giao Tiếp Trong Tổ Chức – Quản Trị Hành Vi
Giao Tiếp Trong Tổ Chức – Quản Trị Hành Vi Giao Tiếp Trong Tổ Chức – Quản Trị Hành Vi
Giao Tiếp Trong Tổ Chức – Quản Trị Hành Vi
 
Phạm trù, Cái chung và Cái riêng
Phạm trù, Cái chung và Cái riêngPhạm trù, Cái chung và Cái riêng
Phạm trù, Cái chung và Cái riêng
 
Đề tài: Nhận thức về vấn đề chủ quyền biển đảo của sinh viên
Đề tài: Nhận thức về vấn đề chủ quyền biển đảo của sinh viên Đề tài: Nhận thức về vấn đề chủ quyền biển đảo của sinh viên
Đề tài: Nhận thức về vấn đề chủ quyền biển đảo của sinh viên
 
200 đề tài khóa luận tốt nghiệp ngành tâm lý học
200 đề tài khóa luận tốt nghiệp ngành tâm lý học200 đề tài khóa luận tốt nghiệp ngành tâm lý học
200 đề tài khóa luận tốt nghiệp ngành tâm lý học
 

Similar to OLYMPIC SINH VIÊN MÔN ĐẠI SỐ ĐỊNH THỨC, HỆ PHƯƠNG TRÌNH TUYẾN TÍNH, MA TRẬN VÀ ÁNH XẠ TUYẾN TÍNH, ĐA THỨC.pdf

bai-giang-thong-ke-xa-hoi-hoc-2022.pdf
bai-giang-thong-ke-xa-hoi-hoc-2022.pdfbai-giang-thong-ke-xa-hoi-hoc-2022.pdf
bai-giang-thong-ke-xa-hoi-hoc-2022.pdf
PhmVnt11
 
Chuyên đề LTĐH
Chuyên đề LTĐH Chuyên đề LTĐH
Chuyên đề LTĐH
Adagio Huynh
 
Chuyen de-luyen-thi-dh-2012
Chuyen de-luyen-thi-dh-2012Chuyen de-luyen-thi-dh-2012
Chuyen de-luyen-thi-dh-2012
Huynh ICT
 
[Www.toan capba.net] chuyen-de-luyen-thi-dh-2012-tran-anh-tuan
[Www.toan capba.net] chuyen-de-luyen-thi-dh-2012-tran-anh-tuan[Www.toan capba.net] chuyen-de-luyen-thi-dh-2012-tran-anh-tuan
[Www.toan capba.net] chuyen-de-luyen-thi-dh-2012-tran-anh-tuan
trongphuckhtn
 
Giáo trình Tester Full
Giáo trình Tester FullGiáo trình Tester Full
Giáo trình Tester Full
Thanh Sơn
 
Đề thi đại học môn Toán theo chủ đề từ 2002 đến 2012
Đề thi đại học môn Toán theo chủ đề từ 2002 đến 2012 Đề thi đại học môn Toán theo chủ đề từ 2002 đến 2012
Đề thi đại học môn Toán theo chủ đề từ 2002 đến 2012
Summer Song
 
[Mathvn.com] tuyen tap de dh 2002-2012 theo chu de
[Mathvn.com] tuyen tap de dh  2002-2012 theo chu de[Mathvn.com] tuyen tap de dh  2002-2012 theo chu de
[Mathvn.com] tuyen tap de dh 2002-2012 theo chu de
hannahisabellla
 
ĐIỀU KHIỂN HỆ ĐA TÁC TỬ.pdf
ĐIỀU KHIỂN HỆ ĐA TÁC TỬ.pdfĐIỀU KHIỂN HỆ ĐA TÁC TỬ.pdf
ĐIỀU KHIỂN HỆ ĐA TÁC TỬ.pdf
Man_Ebook
 

Similar to OLYMPIC SINH VIÊN MÔN ĐẠI SỐ ĐỊNH THỨC, HỆ PHƯƠNG TRÌNH TUYẾN TÍNH, MA TRẬN VÀ ÁNH XẠ TUYẾN TÍNH, ĐA THỨC.pdf (20)

bai-giang-thong-ke-xa-hoi-hoc-2022.pdf
bai-giang-thong-ke-xa-hoi-hoc-2022.pdfbai-giang-thong-ke-xa-hoi-hoc-2022.pdf
bai-giang-thong-ke-xa-hoi-hoc-2022.pdf
 
Chuyên đề LTĐH
Chuyên đề LTĐH Chuyên đề LTĐH
Chuyên đề LTĐH
 
Chuyen de-luyen-thi-dh-2012
Chuyen de-luyen-thi-dh-2012Chuyen de-luyen-thi-dh-2012
Chuyen de-luyen-thi-dh-2012
 
[Www.toan capba.net] chuyen-de-luyen-thi-dh-2012-tran-anh-tuan
[Www.toan capba.net] chuyen-de-luyen-thi-dh-2012-tran-anh-tuan[Www.toan capba.net] chuyen-de-luyen-thi-dh-2012-tran-anh-tuan
[Www.toan capba.net] chuyen-de-luyen-thi-dh-2012-tran-anh-tuan
 
Bài giảng Toán kinh tế
Bài giảng Toán kinh tếBài giảng Toán kinh tế
Bài giảng Toán kinh tế
 
TongHopLyThuyet.pdf
TongHopLyThuyet.pdfTongHopLyThuyet.pdf
TongHopLyThuyet.pdf
 
Chuyên đề tich phan on thi dh
Chuyên đề tich phan on thi dhChuyên đề tich phan on thi dh
Chuyên đề tich phan on thi dh
 
Chuyên đề tich phan on thi dh
Chuyên đề tich phan on thi dhChuyên đề tich phan on thi dh
Chuyên đề tich phan on thi dh
 
Giáo trình Tester Full
Giáo trình Tester FullGiáo trình Tester Full
Giáo trình Tester Full
 
Đề thi đại học môn Toán theo chủ đề từ 2002 đến 2012
Đề thi đại học môn Toán theo chủ đề từ 2002 đến 2012 Đề thi đại học môn Toán theo chủ đề từ 2002 đến 2012
Đề thi đại học môn Toán theo chủ đề từ 2002 đến 2012
 
Dethidaihoc 0266
Dethidaihoc 0266Dethidaihoc 0266
Dethidaihoc 0266
 
Dãy số và giới hạn
Dãy số và giới hạnDãy số và giới hạn
Dãy số và giới hạn
 
Bai_Giang_GT3.pdf
Bai_Giang_GT3.pdfBai_Giang_GT3.pdf
Bai_Giang_GT3.pdf
 
Bài giảng giải tích 3 - thầy nguyễn xuân diệu.pdf
Bài giảng giải tích 3 - thầy nguyễn xuân diệu.pdfBài giảng giải tích 3 - thầy nguyễn xuân diệu.pdf
Bài giảng giải tích 3 - thầy nguyễn xuân diệu.pdf
 
[Mathvn.com] tuyen tap de dh 2002-2012 theo chu de
[Mathvn.com] tuyen tap de dh  2002-2012 theo chu de[Mathvn.com] tuyen tap de dh  2002-2012 theo chu de
[Mathvn.com] tuyen tap de dh 2002-2012 theo chu de
 
Toan a2
Toan a2Toan a2
Toan a2
 
ĐIỀU KHIỂN HỆ ĐA TÁC TỬ.pdf
ĐIỀU KHIỂN HỆ ĐA TÁC TỬ.pdfĐIỀU KHIỂN HỆ ĐA TÁC TỬ.pdf
ĐIỀU KHIỂN HỆ ĐA TÁC TỬ.pdf
 
Chuyen de toan on thi vao 10
Chuyen de toan on thi vao 10Chuyen de toan on thi vao 10
Chuyen de toan on thi vao 10
 
Luận văn: Các dạng phương trình lượng giác, HOT - Gửi miễn phí qua zalo=> 090...
Luận văn: Các dạng phương trình lượng giác, HOT - Gửi miễn phí qua zalo=> 090...Luận văn: Các dạng phương trình lượng giác, HOT - Gửi miễn phí qua zalo=> 090...
Luận văn: Các dạng phương trình lượng giác, HOT - Gửi miễn phí qua zalo=> 090...
 
Luận văn: Các dạng phương trình lượng giác, HAY, 9đ
Luận văn: Các dạng phương trình lượng giác, HAY, 9đLuận văn: Các dạng phương trình lượng giác, HAY, 9đ
Luận văn: Các dạng phương trình lượng giác, HAY, 9đ
 

More from Man_Ebook

More from Man_Ebook (20)

BÀI GIẢNG MÔN HỌC CƠ SỞ NGÔN NGỮ, Dùng cho hệ Cao đẳng chuyên nghiệp.pdf
BÀI GIẢNG MÔN HỌC CƠ SỞ NGÔN NGỮ, Dùng cho hệ Cao đẳng chuyên nghiệp.pdfBÀI GIẢNG MÔN HỌC CƠ SỞ NGÔN NGỮ, Dùng cho hệ Cao đẳng chuyên nghiệp.pdf
BÀI GIẢNG MÔN HỌC CƠ SỞ NGÔN NGỮ, Dùng cho hệ Cao đẳng chuyên nghiệp.pdf
 
TL Báo cáo Thực tập tại Nissan Đà Nẵng.doc
TL Báo cáo Thực tập tại Nissan Đà Nẵng.docTL Báo cáo Thực tập tại Nissan Đà Nẵng.doc
TL Báo cáo Thực tập tại Nissan Đà Nẵng.doc
 
Giáo trình thực vật học 2 - Trường ĐH Cần Thơ.pdf
Giáo trình thực vật học 2 - Trường ĐH Cần Thơ.pdfGiáo trình thực vật học 2 - Trường ĐH Cần Thơ.pdf
Giáo trình thực vật học 2 - Trường ĐH Cần Thơ.pdf
 
Giáo trình mô động vật - Trường ĐH Cần Thơ.pdf
Giáo trình mô động vật - Trường ĐH Cần Thơ.pdfGiáo trình mô động vật - Trường ĐH Cần Thơ.pdf
Giáo trình mô động vật - Trường ĐH Cần Thơ.pdf
 
Giáo trình ngôn ngữ hệ thống A - Trường ĐH Cần Thơ.pdf
Giáo trình ngôn ngữ hệ thống A - Trường ĐH Cần Thơ.pdfGiáo trình ngôn ngữ hệ thống A - Trường ĐH Cần Thơ.pdf
Giáo trình ngôn ngữ hệ thống A - Trường ĐH Cần Thơ.pdf
 
Giáo trình ngôn ngữ mô hình hóa UML - Trường ĐH Cần Thơ.pdf
Giáo trình ngôn ngữ mô hình hóa UML - Trường ĐH Cần Thơ.pdfGiáo trình ngôn ngữ mô hình hóa UML - Trường ĐH Cần Thơ.pdf
Giáo trình ngôn ngữ mô hình hóa UML - Trường ĐH Cần Thơ.pdf
 
Giáo trình nguyên lý máy học - Trường ĐH Cần Thơ.pdf
Giáo trình nguyên lý máy học - Trường ĐH Cần Thơ.pdfGiáo trình nguyên lý máy học - Trường ĐH Cần Thơ.pdf
Giáo trình nguyên lý máy học - Trường ĐH Cần Thơ.pdf
 
Giáo trình mô hình hóa quyết định - Trường ĐH Cần Thơ.pdf
Giáo trình mô hình hóa quyết định - Trường ĐH Cần Thơ.pdfGiáo trình mô hình hóa quyết định - Trường ĐH Cần Thơ.pdf
Giáo trình mô hình hóa quyết định - Trường ĐH Cần Thơ.pdf
 
Giáo trình Linux và phần mềm nguồn mở.pdf
Giáo trình Linux và phần mềm nguồn mở.pdfGiáo trình Linux và phần mềm nguồn mở.pdf
Giáo trình Linux và phần mềm nguồn mở.pdf
 
Giáo trình logic học đại cương - Trường ĐH Cần Thơ.pdf
Giáo trình logic học đại cương - Trường ĐH Cần Thơ.pdfGiáo trình logic học đại cương - Trường ĐH Cần Thơ.pdf
Giáo trình logic học đại cương - Trường ĐH Cần Thơ.pdf
 
Giáo trình lý thuyết điều khiển tự động.pdf
Giáo trình lý thuyết điều khiển tự động.pdfGiáo trình lý thuyết điều khiển tự động.pdf
Giáo trình lý thuyết điều khiển tự động.pdf
 
Giáo trình mạng máy tính - Trường ĐH Cần Thơ.pdf
Giáo trình mạng máy tính - Trường ĐH Cần Thơ.pdfGiáo trình mạng máy tính - Trường ĐH Cần Thơ.pdf
Giáo trình mạng máy tính - Trường ĐH Cần Thơ.pdf
 
Giáo trình lý thuyết xếp hàng và ứng dụng đánh giá hệ thống.pdf
Giáo trình lý thuyết xếp hàng và ứng dụng đánh giá hệ thống.pdfGiáo trình lý thuyết xếp hàng và ứng dụng đánh giá hệ thống.pdf
Giáo trình lý thuyết xếp hàng và ứng dụng đánh giá hệ thống.pdf
 
Giáo trình lập trình cho thiết bị di động.pdf
Giáo trình lập trình cho thiết bị di động.pdfGiáo trình lập trình cho thiết bị di động.pdf
Giáo trình lập trình cho thiết bị di động.pdf
 
Giáo trình lập trình web - Trường ĐH Cần Thơ.pdf
Giáo trình lập trình web  - Trường ĐH Cần Thơ.pdfGiáo trình lập trình web  - Trường ĐH Cần Thơ.pdf
Giáo trình lập trình web - Trường ĐH Cần Thơ.pdf
 
Giáo trình lập trình .Net - Trường ĐH Cần Thơ.pdf
Giáo trình lập trình .Net  - Trường ĐH Cần Thơ.pdfGiáo trình lập trình .Net  - Trường ĐH Cần Thơ.pdf
Giáo trình lập trình .Net - Trường ĐH Cần Thơ.pdf
 
Giáo trình lập trình song song - Trường ĐH Cần Thơ.pdf
Giáo trình lập trình song song  - Trường ĐH Cần Thơ.pdfGiáo trình lập trình song song  - Trường ĐH Cần Thơ.pdf
Giáo trình lập trình song song - Trường ĐH Cần Thơ.pdf
 
Giáo trình lập trình hướng đối tượng.pdf
Giáo trình lập trình hướng đối tượng.pdfGiáo trình lập trình hướng đối tượng.pdf
Giáo trình lập trình hướng đối tượng.pdf
 
Giáo trình lập trình hướng đối tượng Java.pdf
Giáo trình lập trình hướng đối tượng Java.pdfGiáo trình lập trình hướng đối tượng Java.pdf
Giáo trình lập trình hướng đối tượng Java.pdf
 
Giáo trình kỹ thuật phản ứng - Trường ĐH Cần Thơ.pdf
Giáo trình kỹ thuật phản ứng  - Trường ĐH Cần Thơ.pdfGiáo trình kỹ thuật phản ứng  - Trường ĐH Cần Thơ.pdf
Giáo trình kỹ thuật phản ứng - Trường ĐH Cần Thơ.pdf
 

Recently uploaded

SLIDE - Tu van, huong dan cong tac tuyen sinh-2024 (đầy đủ chi tiết).pdf
SLIDE - Tu van, huong dan cong tac tuyen sinh-2024 (đầy đủ chi tiết).pdfSLIDE - Tu van, huong dan cong tac tuyen sinh-2024 (đầy đủ chi tiết).pdf
SLIDE - Tu van, huong dan cong tac tuyen sinh-2024 (đầy đủ chi tiết).pdf
hoangtuansinh1
 

Recently uploaded (20)

Đề cương môn giải phẫu......................
Đề cương môn giải phẫu......................Đề cương môn giải phẫu......................
Đề cương môn giải phẫu......................
 
Giới thiệu Dự án Sản Phụ Khoa - Y Học Cộng Đồng
Giới thiệu Dự án Sản Phụ Khoa - Y Học Cộng ĐồngGiới thiệu Dự án Sản Phụ Khoa - Y Học Cộng Đồng
Giới thiệu Dự án Sản Phụ Khoa - Y Học Cộng Đồng
 
PHƯƠNG THỨC VẬN TẢI ĐƯỜNG SẮT TRONG VẬN TẢI
PHƯƠNG THỨC VẬN TẢI ĐƯỜNG SẮT TRONG VẬN TẢIPHƯƠNG THỨC VẬN TẢI ĐƯỜNG SẮT TRONG VẬN TẢI
PHƯƠNG THỨC VẬN TẢI ĐƯỜNG SẮT TRONG VẬN TẢI
 
SLIDE - Tu van, huong dan cong tac tuyen sinh-2024 (đầy đủ chi tiết).pdf
SLIDE - Tu van, huong dan cong tac tuyen sinh-2024 (đầy đủ chi tiết).pdfSLIDE - Tu van, huong dan cong tac tuyen sinh-2024 (đầy đủ chi tiết).pdf
SLIDE - Tu van, huong dan cong tac tuyen sinh-2024 (đầy đủ chi tiết).pdf
 
GIÁO ÁN DẠY THÊM (KẾ HOẠCH BÀI DẠY BUỔI 2) - TIẾNG ANH 7 GLOBAL SUCCESS (2 CỘ...
GIÁO ÁN DẠY THÊM (KẾ HOẠCH BÀI DẠY BUỔI 2) - TIẾNG ANH 7 GLOBAL SUCCESS (2 CỘ...GIÁO ÁN DẠY THÊM (KẾ HOẠCH BÀI DẠY BUỔI 2) - TIẾNG ANH 7 GLOBAL SUCCESS (2 CỘ...
GIÁO ÁN DẠY THÊM (KẾ HOẠCH BÀI DẠY BUỔI 2) - TIẾNG ANH 7 GLOBAL SUCCESS (2 CỘ...
 
30 ĐỀ PHÁT TRIỂN THEO CẤU TRÚC ĐỀ MINH HỌA BGD NGÀY 22-3-2024 KỲ THI TỐT NGHI...
30 ĐỀ PHÁT TRIỂN THEO CẤU TRÚC ĐỀ MINH HỌA BGD NGÀY 22-3-2024 KỲ THI TỐT NGHI...30 ĐỀ PHÁT TRIỂN THEO CẤU TRÚC ĐỀ MINH HỌA BGD NGÀY 22-3-2024 KỲ THI TỐT NGHI...
30 ĐỀ PHÁT TRIỂN THEO CẤU TRÚC ĐỀ MINH HỌA BGD NGÀY 22-3-2024 KỲ THI TỐT NGHI...
 
kinh tế chính trị mác lênin chương hai và hàng hoá và sxxhh
kinh tế chính trị mác lênin chương hai và hàng hoá và sxxhhkinh tế chính trị mác lênin chương hai và hàng hoá và sxxhh
kinh tế chính trị mác lênin chương hai và hàng hoá và sxxhh
 
powerpoint mẫu họp phụ huynh cuối kì 2 học sinh lớp 7 bgs
powerpoint mẫu họp phụ huynh cuối kì 2 học sinh lớp 7 bgspowerpoint mẫu họp phụ huynh cuối kì 2 học sinh lớp 7 bgs
powerpoint mẫu họp phụ huynh cuối kì 2 học sinh lớp 7 bgs
 
TÀI LIỆU BỒI DƯỠNG HỌC SINH GIỎI LÝ LUẬN VĂN HỌC NĂM HỌC 2023-2024 - MÔN NGỮ ...
TÀI LIỆU BỒI DƯỠNG HỌC SINH GIỎI LÝ LUẬN VĂN HỌC NĂM HỌC 2023-2024 - MÔN NGỮ ...TÀI LIỆU BỒI DƯỠNG HỌC SINH GIỎI LÝ LUẬN VĂN HỌC NĂM HỌC 2023-2024 - MÔN NGỮ ...
TÀI LIỆU BỒI DƯỠNG HỌC SINH GIỎI LÝ LUẬN VĂN HỌC NĂM HỌC 2023-2024 - MÔN NGỮ ...
 
TÀI LIỆU BỒI DƯỠNG HỌC SINH GIỎI KỸ NĂNG VIẾT ĐOẠN VĂN NGHỊ LUẬN XÃ HỘI 200 C...
TÀI LIỆU BỒI DƯỠNG HỌC SINH GIỎI KỸ NĂNG VIẾT ĐOẠN VĂN NGHỊ LUẬN XÃ HỘI 200 C...TÀI LIỆU BỒI DƯỠNG HỌC SINH GIỎI KỸ NĂNG VIẾT ĐOẠN VĂN NGHỊ LUẬN XÃ HỘI 200 C...
TÀI LIỆU BỒI DƯỠNG HỌC SINH GIỎI KỸ NĂNG VIẾT ĐOẠN VĂN NGHỊ LUẬN XÃ HỘI 200 C...
 
cac-cau-noi-tthcm.pdf-cac-cau-noi-tthcm-
cac-cau-noi-tthcm.pdf-cac-cau-noi-tthcm-cac-cau-noi-tthcm.pdf-cac-cau-noi-tthcm-
cac-cau-noi-tthcm.pdf-cac-cau-noi-tthcm-
 
1 - MÃ LỖI SỬA CHỮA BOARD MẠCH BẾP TỪ.pdf
1 - MÃ LỖI SỬA CHỮA BOARD MẠCH BẾP TỪ.pdf1 - MÃ LỖI SỬA CHỮA BOARD MẠCH BẾP TỪ.pdf
1 - MÃ LỖI SỬA CHỮA BOARD MẠCH BẾP TỪ.pdf
 
30 ĐỀ PHÁT TRIỂN THEO CẤU TRÚC ĐỀ MINH HỌA BGD NGÀY 22-3-2024 KỲ THI TỐT NGHI...
30 ĐỀ PHÁT TRIỂN THEO CẤU TRÚC ĐỀ MINH HỌA BGD NGÀY 22-3-2024 KỲ THI TỐT NGHI...30 ĐỀ PHÁT TRIỂN THEO CẤU TRÚC ĐỀ MINH HỌA BGD NGÀY 22-3-2024 KỲ THI TỐT NGHI...
30 ĐỀ PHÁT TRIỂN THEO CẤU TRÚC ĐỀ MINH HỌA BGD NGÀY 22-3-2024 KỲ THI TỐT NGHI...
 
3-BẢNG MÃ LỖI CỦA CÁC HÃNG ĐIỀU HÒA .pdf - ĐIỆN LẠNH BÁCH KHOA HÀ NỘI
3-BẢNG MÃ LỖI CỦA CÁC HÃNG ĐIỀU HÒA .pdf - ĐIỆN LẠNH BÁCH KHOA HÀ NỘI3-BẢNG MÃ LỖI CỦA CÁC HÃNG ĐIỀU HÒA .pdf - ĐIỆN LẠNH BÁCH KHOA HÀ NỘI
3-BẢNG MÃ LỖI CỦA CÁC HÃNG ĐIỀU HÒA .pdf - ĐIỆN LẠNH BÁCH KHOA HÀ NỘI
 
Danh sách sinh viên tốt nghiệp Đại học - Cao đẳng Trường Đại học Phú Yên năm ...
Danh sách sinh viên tốt nghiệp Đại học - Cao đẳng Trường Đại học Phú Yên năm ...Danh sách sinh viên tốt nghiệp Đại học - Cao đẳng Trường Đại học Phú Yên năm ...
Danh sách sinh viên tốt nghiệp Đại học - Cao đẳng Trường Đại học Phú Yên năm ...
 
Campbell _2011_ - Sinh học - Tế bào - Ref.pdf
Campbell _2011_ - Sinh học - Tế bào - Ref.pdfCampbell _2011_ - Sinh học - Tế bào - Ref.pdf
Campbell _2011_ - Sinh học - Tế bào - Ref.pdf
 
Kiểm tra cuối học kì 1 sinh học 12 đề tham khảo
Kiểm tra cuối học kì 1 sinh học 12 đề tham khảoKiểm tra cuối học kì 1 sinh học 12 đề tham khảo
Kiểm tra cuối học kì 1 sinh học 12 đề tham khảo
 
GIÁO TRÌNH KHỐI NGUỒN CÁC LOẠI - ĐIỆN LẠNH BÁCH KHOA HÀ NỘI
GIÁO TRÌNH  KHỐI NGUỒN CÁC LOẠI - ĐIỆN LẠNH BÁCH KHOA HÀ NỘIGIÁO TRÌNH  KHỐI NGUỒN CÁC LOẠI - ĐIỆN LẠNH BÁCH KHOA HÀ NỘI
GIÁO TRÌNH KHỐI NGUỒN CÁC LOẠI - ĐIỆN LẠNH BÁCH KHOA HÀ NỘI
 
sách sinh học đại cương - Textbook.pdf
sách sinh học đại cương   -   Textbook.pdfsách sinh học đại cương   -   Textbook.pdf
sách sinh học đại cương - Textbook.pdf
 
BỘ LUYỆN NGHE VÀO 10 TIẾNG ANH DẠNG TRẮC NGHIỆM 4 CÂU TRẢ LỜI - CÓ FILE NGHE.pdf
BỘ LUYỆN NGHE VÀO 10 TIẾNG ANH DẠNG TRẮC NGHIỆM 4 CÂU TRẢ LỜI - CÓ FILE NGHE.pdfBỘ LUYỆN NGHE VÀO 10 TIẾNG ANH DẠNG TRẮC NGHIỆM 4 CÂU TRẢ LỜI - CÓ FILE NGHE.pdf
BỘ LUYỆN NGHE VÀO 10 TIẾNG ANH DẠNG TRẮC NGHIỆM 4 CÂU TRẢ LỜI - CÓ FILE NGHE.pdf
 

OLYMPIC SINH VIÊN MÔN ĐẠI SỐ ĐỊNH THỨC, HỆ PHƯƠNG TRÌNH TUYẾN TÍNH, MA TRẬN VÀ ÁNH XẠ TUYẾN TÍNH, ĐA THỨC.pdf

  • 1. TRƯỜNG ĐẠI HỌC BÁCH KHOA HÀ NỘI VIỆN TOÁN ỨNG DỤNG & TIN HỌC BÙI XUÂN DIỆU Bài Giảng OLYMPIC SINH VIÊN MÔN ĐẠI SỐ ĐỊNH THỨC, HỆ PHƯƠNG TRÌNH TUYẾN TÍNH, MA TRẬN VÀ ÁNH XẠ TUYẾN TÍNH, ĐA THỨC Tóm tắt lý thuyết, các ví dụ, bài tập và lời giải Dresden (Germany) - 2012
  • 2. MỤC LỤC Mục lục. . . . . . . . . . . . . . . . . . . . . . . . . . . . . . . 1 Chương 1 . Ma trận - Định thức . . . . . . . . . . . . . . . . . . . . . . . 5 1 Định thức . . . . . . . . . . . . . . . . . . . . . . . . . . . . . . . . . . . . . . . 5 1.1 Các tính chất cơ bản của định thức . . . . . . . . . . . . . . . . . . . . 5 1.2 Các định thức đặc biệt . . . . . . . . . . . . . . . . . . . . . . . . . . . 6 1.3 Bài tập . . . . . . . . . . . . . . . . . . . . . . . . . . . . . . . . . . . . 11 2 Định thức con và phần phụ đại số . . . . . . . . . . . . . . . . . . . . . . . . . 13 2.1 Các định nghĩa và tính chất . . . . . . . . . . . . . . . . . . . . . . . . 13 2.2 Bài tập . . . . . . . . . . . . . . . . . . . . . . . . . . . . . . . . . . . . 14 3 Phần bù Schur . . . . . . . . . . . . . . . . . . . . . . . . . . . . . . . . . . . . 16 3.1 Các định nghĩa và tính chất . . . . . . . . . . . . . . . . . . . . . . . . 16 3.2 Bài tập . . . . . . . . . . . . . . . . . . . . . . . . . . . . . . . . . . . . 17 Chương 2 . Không gian véctơ - Ánh xạ tuyến tính . . . . . . . . . . . . . . 19 1 Không gian đối ngẫu - Phần bù trực giao . . . . . . . . . . . . . . . . . . . . . 19 1.1 Không gian đối ngẫu . . . . . . . . . . . . . . . . . . . . . . . . . . . . 19 1.2 Phần bù trực giao . . . . . . . . . . . . . . . . . . . . . . . . . . . . . . 21 1.3 Bài tập . . . . . . . . . . . . . . . . . . . . . . . . . . . . . . . . . . . . 21 2 Hạt nhân và ảnh - Không gian thương . . . . . . . . . . . . . . . . . . . . . . 22 2.1 Hạt nhân và ảnh . . . . . . . . . . . . . . . . . . . . . . . . . . . . . . 22 2.2 Không gian thương . . . . . . . . . . . . . . . . . . . . . . . . . . . . . 23 2.3 Bài tập . . . . . . . . . . . . . . . . . . . . . . . . . . . . . . . . . . . . 24 3 Cơ sở của không gian véctơ - Độc lập tuyến tính . . . . . . . . . . . . . . . . . 25 3.1 Bài toán đổi cơ sở . . . . . . . . . . . . . . . . . . . . . . . . . . . . . . 25 3.2 Bài tập . . . . . . . . . . . . . . . . . . . . . . . . . . . . . . . . . . . . 25 4 Hạng của ma trận . . . . . . . . . . . . . . . . . . . . . . . . . . . . . . . . . . 27 4.1 Các tính chất của hạng của ma trận . . . . . . . . . . . . . . . . . . . 27 4.2 Bài tập . . . . . . . . . . . . . . . . . . . . . . . . . . . . . . . . . . . . 28 1
  • 3. 2 MỤC LỤC Chương 3 . Dạng chính tắc của ma trận và toán tử tuyến tính . . . . . . . . 31 1 Vết của ma trận . . . . . . . . . . . . . . . . . . . . . . . . . . . . . . . . . . . 31 2 Cấu trúc của tự đồng cấu . . . . . . . . . . . . . . . . . . . . . . . . . . . . . . 32 2.1 Trị riêng và véctơ riêng . . . . . . . . . . . . . . . . . . . . . . . . . . . 32 2.2 Tự đồng cấu chéo hoá được . . . . . . . . . . . . . . . . . . . . . . . . . 33 2.3 Đa thức tối tiểu . . . . . . . . . . . . . . . . . . . . . . . . . . . . . . . 34 2.4 Bài tập . . . . . . . . . . . . . . . . . . . . . . . . . . . . . . . . . . . . 35 3 Dạng chuẩn của ma trận . . . . . . . . . . . . . . . . . . . . . . . . . . . . . . 40 3.1 Dạng chuẩn Jordan của ma trận . . . . . . . . . . . . . . . . . . . . . 40 3.2 Dạng chuẩn Frobenius . . . . . . . . . . . . . . . . . . . . . . . . . . . 41 3.3 Bài tập . . . . . . . . . . . . . . . . . . . . . . . . . . . . . . . . . . . . 42 4 Biểu diễn ma trận . . . . . . . . . . . . . . . . . . . . . . . . . . . . . . . . . . 43 4.1 Rút gọn một ma trận về ma trận dạng đường chéo đơn giản . . . . . . 43 4.2 Biểu ma trận dưới dạng tọa độ cực . . . . . . . . . . . . . . . . . . . . 43 4.3 Biểu diễn Schur . . . . . . . . . . . . . . . . . . . . . . . . . . . . . . . 44 4.4 Biểu diễn Lanczos . . . . . . . . . . . . . . . . . . . . . . . . . . . . . . 44 4.5 Bài tập . . . . . . . . . . . . . . . . . . . . . . . . . . . . . . . . . . . . 44 Chương 4 . Các ma trận có dạng đặc biệt . . . . . . . . . . . . . . . . . . 45 1 Ma trận đối xứng - Ma trận Hermitian . . . . . . . . . . . . . . . . . . . . . . 45 1.1 Các định nghĩa và tính chất . . . . . . . . . . . . . . . . . . . . . . . . 45 1.2 Bài tập . . . . . . . . . . . . . . . . . . . . . . . . . . . . . . . . . . . . 46 2 Ma trận phản xứng . . . . . . . . . . . . . . . . . . . . . . . . . . . . . . . . . 47 2.1 Các định nghĩa và tính chất . . . . . . . . . . . . . . . . . . . . . . . . 47 2.2 Bài tập . . . . . . . . . . . . . . . . . . . . . . . . . . . . . . . . . . . . 47 3 Ma trận trực giao - Phép biến đổi Cayley . . . . . . . . . . . . . . . . . . . . . 48 3.1 Các định nghĩa và tính chất . . . . . . . . . . . . . . . . . . . . . . . . 48 3.2 Bài tập . . . . . . . . . . . . . . . . . . . . . . . . . . . . . . . . . . . . 48 4 Ma trận chuẩn tắc . . . . . . . . . . . . . . . . . . . . . . . . . . . . . . . . . . 50 4.1 Các định nghĩa và tính chất . . . . . . . . . . . . . . . . . . . . . . . . 50 4.2 Bài tập . . . . . . . . . . . . . . . . . . . . . . . . . . . . . . . . . . . . 50 5 Ma trận luỹ linh . . . . . . . . . . . . . . . . . . . . . . . . . . . . . . . . . . . 52 5.1 Các định nghĩa và tính chất . . . . . . . . . . . . . . . . . . . . . . . . 52 5.2 Bài tập . . . . . . . . . . . . . . . . . . . . . . . . . . . . . . . . . . . . 52 6 Toán tử chiếu - Ma trận lũy đẳng . . . . . . . . . . . . . . . . . . . . . . . . . 54 6.1 Các định nghĩa và tính chất . . . . . . . . . . . . . . . . . . . . . . . . 54 6.2 Bài tập . . . . . . . . . . . . . . . . . . . . . . . . . . . . . . . . . . . . 54 7 Ma trận đối hợp . . . . . . . . . . . . . . . . . . . . . . . . . . . . . . . . . . . 57
  • 4. MỤC LỤC 3 8 Ma trận hoán vị (hay còn gọi là ma trận giao hoán) . . . . . . . . . . . . . . . 58 8.1 Định nghĩa . . . . . . . . . . . . . . . . . . . . . . . . . . . . . . . . . . 58 8.2 Bài tập . . . . . . . . . . . . . . . . . . . . . . . . . . . . . . . . . . . . 58 Chương 5 . Các bất đẳng thức ma trận. . . . . . . . . . . . . . . . . . . . 59 1 Các bất đẳng thức cho ma trận đối xứng và Hermitian . . . . . . . . . . . . . 59 1.1 Các định lý cơ bản . . . . . . . . . . . . . . . . . . . . . . . . . . . . . . 59 1.2 Bài tập . . . . . . . . . . . . . . . . . . . . . . . . . . . . . . . . . . . . 60 2 Các bất đẳng thức cho trị riêng . . . . . . . . . . . . . . . . . . . . . . . . . . 61 2.1 Các bất đẳng thức cơ bản . . . . . . . . . . . . . . . . . . . . . . . . . . 61 2.2 Bài tập . . . . . . . . . . . . . . . . . . . . . . . . . . . . . . . . . . . . 62 Chương 6 . Đa thức . . . . . . . . . . . . . . . . . . . . . . . . . . . . . 63
  • 6. CHƯƠNG 1 MA TRẬN - ĐỊNH THỨC §1. ĐỊNH THỨC 1.1 Các tính chất cơ bản của định thức Định thức của một ma trận vuông A = (aij)n 1 cấp n là tổng luân phiên ∑ σ (−1)σ a1σ(1)a2σ(2) . . . anσ(n), ở đó tổng trên được lấy qua tất cả các phép hoán vị σ ∈ Sn. Định thức của ma trận A được kí hiệu là det A hoặc |A|, nếu det A 6= 0 ta nói A là ma trận khả nghịch (không suy biến). Các tính chất sau đây thường được sử dụng để tính định thức của một ma trận. Các bạn có thể kiểm chứng hoặc chứng minh chúng một cách dễ dàng. 1. Nếu đổi chỗ hai hàng (hoặc hai cột) của ma trận A thì định thức của nó đổi dấu. Nói riêng, nếu ma trận A có hai hàng (cột)giống nhau thì det A = 0. 2. Nếu A, B và C là các ma trận vuông cùng cấp thì det A C 0 B ! = det A. det B. 3. det A = n ∑ j=1 (−1)i+jMi,j, ở đó Mij là định thức của ma trận thu được từ A bằng cách bỏ đi hàng thứ i và cột thứ j của nó. Công thức này còn được gọi là công thức khai triển định thức theo hàng. Các bạn có thể tự viết công thức khai triển định thức theo cột một cách tương tự. 4.
  • 7.
  • 8.
  • 9.
  • 10.
  • 11.
  • 12.
  • 13. λ1α1 + µ1β1 a12 . . . a1n . . . . . . . . . . . . λnαn + µnβn an2 . . . ann
  • 14.
  • 15.
  • 16.
  • 17.
  • 18.
  • 19.
  • 20. = λ
  • 21.
  • 22.
  • 23.
  • 24.
  • 25.
  • 26.
  • 27. α1 a12 . . . a1n . . . . . . . . . . . . αn an2 . . . ann
  • 28.
  • 29.
  • 30.
  • 31.
  • 32.
  • 33.
  • 34. + µ
  • 35.
  • 36.
  • 37.
  • 38.
  • 39.
  • 40.
  • 41. β1 a12 . . . a1n . . . . . . . . . . . . βn an2 . . . ann
  • 42.
  • 43.
  • 44.
  • 45.
  • 46.
  • 47.
  • 48. 5
  • 49. 6 Chương 1. Ma trận - Định thức 5. det(AB) = det A det B. 6. det(AT) = detA. 1.2 Các định thức đặc biệt Định thức Vandermonde Ma trận Vandermonde cấp n là ma trận vuông cấp n có dạng Vn(a1, a2, ..., an) =         1 1 . . . 1 1 a1 a2 . . . an−1 an a2 1 a2 2 . . . a2 n−1 a2 n . . . . . . ... . . . . . . an−1 1 an−1 2 . . . an−1 n−1 an−1 n         Định lý 1.1. Chứng minh rằng det Vn(a1, a2, ..., an) = ∏ 16i<j6n (aj − ai). Từ đó suy ra hệ Vn(a1, a2, ..., an).X = 0 chỉ có nghiệm tầm thường khi và chỉ khi a1, a2, . . . , an đôi một phân biệt. Một ứng dụng thú vị của định thức Vandermonde là bài toán sau: Bài tập 1.1. Cho A là một ma trận vuông cấp n. Khi đó An = 0 ⇔ tr(Ak ) = 0, k = 0, 1, 2, . . . , n Chứng minh. ⇒ Nếu An = 0 thì A là một ma trận lũy linh, do đó A chỉ có các trị riêng bằng 0, nên Ak cũng chỉ có các trị riêng bằng 0 với mọi k. Suy ra điều phải chứng minh. ⇐ Giả sử các giá trị riêng của A là λ1, λ2, . . . , λn. Khi đó từ tr(Ak) = 0, k = 0, 1, 2, . . . , n ta có hệ phương trình:                λ1 + λ2 + . . . + λn = 0 λ2 1 + λ2 2 + . . . + λ2 n = 0 . . . λn 1 + λn 2 + . . . + λn n = 0 (1.1) hay Vn(λ1, λ2, . . . , λn)(λ1, λ2, . . . , λn)T = 0. Ta sẽ chứng minh tất cả các giá trị riêng của A bằng nhau. Thật vậy: Nếu λi đôi một phân biệt thì định thức Vandermonde khác không, hệ phương trình trên chỉ có nghiệm duy nhất λ1, λ2, . . . , λn = 0. Mâu thuẫn.
  • 50. 1. Định thức 7 Ngược lại, không mất tính tổng quát, giả sử λ1 = λ2 và không một giá trị λi còn lại nào bằng nhau. Khi đó hệ phương trình được viết lại dưới dạng Vn−1(λ2, . . . , λn)(2λ2, . . . , λn)T = 0 Lập luận tương tự ta có λ2 = . . . = λn = 0, mâu thuẫn. Vậy tất cả các trị riêng của A bằng nhau và do đó bằng 0. Bài tập 1.2. Chứng minh rằng với các số nguyên k1 < k2 < ... < kn bất kì thì det Vn(k1, k2, ..., kn) det Vn(1, 2, ..., n) là một số nguyên. Bài tập 1.3. Cho W là ma trận có được từ ma trận V = Vn(a1, a2, ..., an) bằng cách thay hàng (an−1 1 , an−1 2 , ..., an−1 n ) bởi hàng (an 1 , an 2 , ..., an n). Chứng minh rằng det W = (a1 + a2 + ... + an) det V. Bài tập 1.4. Chứng minh rằng det          1 1 . . . 1 1 a1 a2 . . . an−1 an . . . . . . ... . . . . . . an−2 1 an−2 2 . . . an−2 n−1 an−2 n a2a3...an a1a3...an . . . a1a2...an−2an a1a2...an−1          = (−1)n−1 . det Vn(a1, a2, ..., an) Chứng minh. • Nếu a1, a2, . . . , an 6= 0 thì nhân cột thứ nhất với a1, cột thứ hai với a2, . . . , cột thứ n với an rồi chia cho a1a2 . . . an ta được det          1 1 . . . 1 1 a1 a2 . . . an−1 an . . . . . . ... . . . . . . an−2 1 an−2 2 . . . an−2 n−1 an−2 n a2a3...an a1a3...an . . . a1a2...an−2an a1a2...an−1          = 1 a1a2 . . . an . det          a1 a2 . . . an−1 an a2 1 a2 2 . . . a2 n−1 a2 n . . . . . . ... . . . . . . an−1 1 an−1 2 . . . an−1 n−1 an−1 n 1 1 . . . 1 1          = (−1)n−1. det Vn(a1, a2, ..., an) • Trường hợp có ít nhất một trong các số a1, a2, . . . , an bằng 0 (xét riêng).
  • 51. 8 Chương 1. Ma trận - Định thức Bài tập 1.5. Cho f1(x), f2(x), ..., fn(x) là các đa thức bậc không quá n − 2. Chứng minh rằng với mọi số a1, a2, . . . , an ta có
  • 52.
  • 53.
  • 54.
  • 55.
  • 56.
  • 57.
  • 58.
  • 59.
  • 60.
  • 61. f1(a1) f1(a2) . . . f1(an) f2(a1) f2(a2) . . . f2(an) . . . . . . ... . . . fn(a1) fn(a2) . . . fn(an)
  • 62.
  • 63.
  • 64.
  • 65.
  • 66.
  • 67.
  • 68.
  • 69.
  • 70.
  • 71. = 0 Chứng minh. Giả sử fi(x) = bi0 + bi1x + . . . + bi,n−2xn−2 thì       f1(a1) f1(a2) . . . f1(an) f2(a1) f2(a2) . . . f2(an) . . . . . . ... . . . fn(a1) fn(a2) . . . fn(an)       =       b10 b11 . . . b1,n−2 0 b20 b21 . . . b2,n−2 0 . . . . . . ... . . . . . . bn0 bn1 . . . bn,n−2 0       .       1 1 . . . 1 1 a1 a2 . . . an−1 an . . . . . . ... . . . . . . an−1 1 an−1 2 . . . an−1 n−1 an−1 n       Từ đó ta có điều phải chứng minh. Bài tập 1.6. Cho A = aij và fi(x) = a1i + a2ix + ... + anixn−1 với i = 1, n. Chứng minh rằng
  • 72.
  • 73.
  • 74.
  • 75.
  • 76.
  • 77.
  • 78.
  • 79.
  • 80.
  • 81. f1(x1) f1(x2) . . . f1(xn) f2(x1) f2(x2) . . . f2(xn) . . . . . . ... . . . fn(x1) fn(x2) . . . fn(xn)
  • 82.
  • 83.
  • 84.
  • 85.
  • 86.
  • 87.
  • 88.
  • 89.
  • 90.
  • 91. = det A.Vn(x1, x2, ..., xn) Chứng minh. Tương tự như bài ?? ta có       f1(x1) f1(x2) . . . f1(xn) f2(x1) f2(x2) . . . f2(xn) . . . . . . ... . . . fn(x1) fn(x2) . . . fn(xn)       =       a11 a12 . . . a1,n−1 a1n a21 a22 . . . a2,n−1 a2n . . . . . . ... . . . . . . an1 an2 . . . an,n−1 ann       .       1 1 . . . 1 1 x1 x2 . . . xn−1 xn . . . . . . ... . . . . . . xn−1 1 xn−1 2 . . . xn−1 n−1 xn−1 n       Suy ra điều phải chứng minh. Bài tập 1.7. Chứng minh rằng với k1, k2, . . . , kn là các số tự nhiện khác nhau và a1, a2, . . . , an là các số dương khác nhau thì det         1 1 1 . . . 1 ak1 1 ak1 2 ak1 3 . . . ak1 n ak2 1 ak2 2 ak2 3 . . . ak2 n . . . . . . . . . . . . . . . akn 1 akn 2 akn 3 . . . akn n         6= 0
  • 92. 1. Định thức 9 Định thức Cauchy Ma trận Cauchy là ma trận vuông cấp n, A = (aij), ở đó aij = 1 xi+yj . Bằng phương pháp quy nạp, ta sẽ chứng minh det A = Πij(xi − xj)(yi − yj) Πi,j(xi + xj) Trước hết lấy mỗi cột từ 1 đến n − 1 trừ đi cột cuối cùng, ta được a0 ij = (xi + yj)−1 − (xi + yn)−1 = (yn − yj)(xi + yn)−1 (xi + yj)−1 với j 6= n. Đưa nhân tử (xi + yn)−1 ở mỗi hàng, và yn − yj ở mỗi cột trừ cột cuối cùng ra khỏi định thức ta sẽ thu được định thức |bij|n i,j=1, ở đó bij = aij với j 6= n và bin = 1. Tiếp theo, lấy mỗi hàng từ 1 đến n − 1 trừ đi hàng cuối cùng. Đưa nhân tử xn − xi ở mỗi hàng trừ hàng cuối cùng, và nhân tử (xn + yj)−1 ở mỗi cột trừ cột cuối cùng, ta sẽ thu được công thức truy hồi định thức Cauchy cấp n qua cấp n − 1. Định thức Frobenius Ma trận có dạng            0 1 0 . . . 0 0 0 0 1 . . . 0 0 . . . . . . ... ... ... . . . 0 0 0 . . . 1 0 0 0 0 . . . 0 1 a0 a1 a2 . . . an−2 an−1            được gọi là ma trận Frobenius, hay ma trận bạn của đa thức p(λ) = λn − an−1λn−1 − an−2λn−2 − . . . − a0. Khai triển định thức Frobenius theo hàng thứ nhất, các bạn có thể dễ dàng thu được công thức sau: det(λI − A) = p(λ)
  • 93. 10 Chương 1. Ma trận - Định thức Định thức của ma trận ba đường chéo Ma trận ba đường chéo là ma trận vuông J = (aij)n i,j=1, ở đó aij = 0 với |i − j| 1. Đặt ai = aii, bi = ai,i+1, ci = ai+1,i, ma trận ba đường chéo khi đó có dạng:               a1 b1 0 . . . 0 0 0 c1 a2 b2 . . . 0 0 0 0 c2 a3 ... 0 0 0 . . . . . . ... ... ... . . . . . . 0 0 0 ... an−2 bn−2 0 0 0 0 . . . cn−2 an−1 bn−1 0 0 0 . . . 0 cn−1 an               Khai triển định thức của ma trận trên theo hàng thứ k, ta được ∆k = ak∆k−1 − bk−1ck∆k−2 với k ≥ 2, ở đó ∆k = det(aij)k i,j=1. Công thức truy hồi trên đã khẳng định rằng định thức ∆n không những chỉ phụ thuộc vào các số bi, cj mà còn phụ thuộc vào bici. Trong trường hợp đặc biệt, kí hiệu (a1 . . . an)
  • 94.
  • 95.
  • 96.
  • 97.
  • 98.
  • 99.
  • 100.
  • 101.
  • 102.
  • 103.
  • 104.
  • 105.
  • 106.
  • 107.
  • 108.
  • 109.
  • 110.
  • 111. a1 1 0 . . . 0 0 0 −1 a2 1 . . . 0 0 0 0 −1 a3 ... 0 0 0 . . . . . . ... ... ... . . . . . . 0 0 0 ... an−2 1 0 0 0 0 . . . −1 an−1 1 0 0 0 . . . 0 −1 an
  • 112.
  • 113.
  • 114.
  • 115.
  • 116.
  • 117.
  • 118.
  • 119.
  • 120.
  • 121.
  • 122.
  • 123.
  • 124.
  • 125.
  • 126.
  • 127.
  • 128.
  • 129. ta có công thức truy hồi thông qua liên phân số sau: (a1a2 . . . an) (a2a3 . . . an) = a1 + 1 a2 + 1 a3+ ...+ 1 an−1+ 1 an Định thức của ma trận khối Giả sử A = A11 A12 A21 A22 ! , ở đó A11 và A22 là các ma trận vuông cấp m và cấp n tương ứng. Đặt D là một ma trận vuông cấp m và B là ma trận cỡ n × m. Định lý 1.2.
  • 130.
  • 131.
  • 132.
  • 133.
  • 135.
  • 136.
  • 137.
  • 138.
  • 140.
  • 141.
  • 142.
  • 143.
  • 144. A11 A12 A21 + BA11 A22 + BA12
  • 145.
  • 146.
  • 147.
  • 148.
  • 149. = |A|.
  • 150. 1. Định thức 11 1.3 Bài tập Bài tập 1.8. Cho A là một ma trận phản xứng cấp n lẻ. Chứng minh rằng det A = 0. Bài tập 1.9. Chứng minh rằng định thức của một ma trận phản xứng cấp n chẵn không thay đổi nếu ta cộng thêm vào mỗi phần tử của nó với một số cố định. Bài tập 1.10. Tính định thức của một ma trận phản xứng cấp 2n chẵn thỏa mãn tính chất các phần tử ở trên đường chéo chính bằng 1. Bài tập 1.11. Cho A = (aij)n i,j=1, với aij = a|i−j|. Tính det A. Bài tập 1.12. Cho ∆3 =
  • 151.
  • 152.
  • 153.
  • 154.
  • 155.
  • 156.
  • 157.
  • 158.
  • 159. 1 −1 0 0 x h −1 0 x2 hx h −1 x3 hx2 hx h
  • 160.
  • 161.
  • 162.
  • 163.
  • 164.
  • 165.
  • 166.
  • 167.
  • 168. và ∆n được định nghĩa tương tự cho n 3. Chứng minh rằng ∆n = (x + h)n. Bài tập 1.13. Cho C = (cij)n i,j=1, với cij =    aibj nếu i 6= j xi nếu i = j . Tính det C. Bài tập 1.14. Cho ai,i+1 = ci với i = 1, . . . , n, các phần tử khác của ma trận A bằng 0. Chứng minh rằng định thức của ma trận I + A + A2 + . . . + An−1 bằng (1 − c)n−1, với c = c1 . . . cn. Bài tập 1.15. Tính det(aij)n i,j=1, với aij = (1 − xiyj)−1. Bài tập 1.16. Tính
  • 169.
  • 170.
  • 171.
  • 172.
  • 173.
  • 174.
  • 175. 1 x1 . . . xn−2 1 (x2 + x3 + . . . + xn)n−1 . . . . . . . . . . . . . . . 1 xn . . . xn−2 n (x1 + x2 + . . . + xn−1)n−1
  • 176.
  • 177.
  • 178.
  • 179.
  • 180.
  • 181.
  • 183.
  • 184.
  • 185.
  • 186.
  • 187.
  • 188.
  • 189. 1 x1 . . . xn−2 1 x2x3 . . . xn . . . . . . . . . . . . . . . 1 xn . . . xn−2 n x1x2 . . . xn−1
  • 190.
  • 191.
  • 192.
  • 193.
  • 194.
  • 195.
  • 196. Bài tập 1.18. Tính |aik|n 0 , với aik = λn−k i (1 + λ2 i )k. Bài tập 1.19. Cho aij = Cin j . Chứng minh rằng |aij|r 1 = nr(r+1)/2 với r ≤ n.
  • 197. 12 Chương 1. Ma trận - Định thức Bài tập 1.20. Cho k1, . . . , kn ∈ Z, tính |aij|n 1 , ở đó aij =    1 (ki+j−i)! với ki + j − i ≥ 0 0 với ki + j − i 0 Bài tập 1.21. Cho sk = p1xk 1 + . . . + pnxk n, và ai,j = si+j. Chứng minh rằng |aij|n−1 0 = p1 . . . pnΠij(xi − xj)2 . Bài tập 1.22. Cho s = xk 1 + . . . + xk n. Tính
  • 198.
  • 199.
  • 200.
  • 201.
  • 202.
  • 203.
  • 204.
  • 205.
  • 206.
  • 207. s0 s1 . . . sn−1 1 s1 s2 . . . sn y . . . . . . . . . . . . . . . sn sn+1 . . . s2n− yn
  • 208.
  • 209.
  • 210.
  • 211.
  • 212.
  • 213.
  • 214.
  • 215.
  • 216.
  • 217. Bài tập 1.23. Cho aij = (xi + yj)n. Chứng minh rằng |aij|n 0 = Cn 1 Cn 2 . . . Cn nΠik(xi − xk)(yk − yi). Bài tập 1.24. Cho bij = (−1)i+jaij. Chứng minh rằng |aij|n 1 = |bij|n 1 . Bài tập 1.25. Cho ∆n(k) = |aij|n 0 , ở đó aij = Ck+i 2j . Chứng minh rằng ∆n(k) = k(k + 1) . . . (k + n − 1) 1.3 . . . (2n − 1) ∆n−1(k − 1). Bài tập 1.26. Cho Dn = |aij|n 0 , ở đó aij = Cn 2j−1. Chứng minh rằng Dn = 2n(n+1)/2. Bài tập 1.27. Cho A = A11 A12 A21 A22 ! và B = B11 B12 B21 B22 ! , ở đó A11, B11, A22, B22 là các ma trận vuông cùng cấp và rank A11 = rank A, rank B11 = rank B. Chứng minh rằng
  • 218.
  • 219.
  • 220.
  • 221.
  • 223.
  • 224.
  • 225.
  • 226.
  • 227. .
  • 228.
  • 229.
  • 230.
  • 231.
  • 233.
  • 234.
  • 235.
  • 236.
  • 237. = |A + B|.|A11|.|B22|
  • 238. 2. Định thức con và phần phụ đại số 13 §2. ĐỊNH THỨC CON VÀ PHẦN PHỤ ĐẠI SỐ 2.1 Các định nghĩa và tính chất Định nghĩa 1.3. Ma trận mà các phần tử của nó là giao của p hàng và p cột của ma trận vuông A được gọi là ma trận con cấp p của A. Định thức tương ứng được gọi là định thức con cấp p. Kí hiệu A i1 . . . ip k1 . . . kp ! =
  • 239.
  • 240.
  • 241.
  • 242.
  • 243.
  • 244.
  • 245.
  • 246. ai1k1 ai1k2 . . . ai1kp . . . . . . . . . . . . aipk1 aipk2 . . . aipkp
  • 247.
  • 248.
  • 249.
  • 250.
  • 251.
  • 252.
  • 253.
  • 254. Nếu i1 = k1, . . . , ip = kp thì định thức con được gọi là định thức con chính cấp p. Định nghĩa 1.4. Định thức con khác 0 có bậc cao nhất được gọi là định thức con cơ sở và cấp của nó được gọi là hạng của ma trận A. Định lý 1.5. Nếu A i1 . . . ip k1 . . . kp ! là một định thức con cơ sở của A, thì các hàng của ma trận A là tổ hợp tuyến tính của các hàng i1, . . . , ip của nó, và các hàng i1, . . . , ip này độc lập tuyến tính. Hệ quả 1.6. Hạng của một ma trận bằng số các hàng (cột) độc lập tuyến tính lớn nhất của nó. Định lý 1.7 (Công thức Binet - Cauchy). Giả sử A và B là các ma trận cỡ n × m và m × n tương ứng và n ≤ m. Khi đó det AB = ∑ 1≤k1k2...kn≤m Ak1...kn Bk1...kn , ở đó Ak1...kn là định thức con thu được từ các cột k1, . . . , kn của A và Bk1...kn là định thức con thu được từ các hàng k1, . . . , kn của B. Kí hiệu Aij = (−1)i+jMij, ở đó Mij là định thức của ma trận thu được từ ma trận A bằng cách bỏ đi hàng thứ i và cột thứ j, nó được gọi là phần bù đại số của phần tử aij. Khi đó ma trận adj A = (Aij)T được gọi là ma trận liên hợp của ma trận A. Ta có công thức sau: A. adj(A) = det A.I Định lý 1.8. Toán tử adj có các tính chất sau:
  • 255. 14 Chương 1. Ma trận - Định thức 1. adj AB = adj B. adj A 2. adj XAX−1 = X(adj A)X−1 3. Nếu AB = BA thì (adj A)B = B(adj A). Định lý 1.9.
  • 256.
  • 257.
  • 258.
  • 259.
  • 260.
  • 261.
  • 262. A11 . . . A1p . . . . . . . . . Ap1 . . . App
  • 263.
  • 264.
  • 265.
  • 266.
  • 267.
  • 268.
  • 270.
  • 271.
  • 272.
  • 273.
  • 274.
  • 275.
  • 276. Ap+1,p+1 . . . Ap+1,n . . . . . . . . . An,p+1 . . . Ann
  • 277.
  • 278.
  • 279.
  • 280.
  • 281.
  • 282.
  • 283. Hệ quả 1.10. Nếu A là ma trận suy biến thì rank(adj A) ≤ 1. Định lý 1.11 (Jacobi). Giả sử 1 ≤ p n và σ = i1 . . . in j1 . . . kn ! là một phép hoán vị bất kì. Khi đó
  • 284.
  • 285.
  • 286.
  • 287.
  • 288.
  • 289.
  • 290.
  • 291. Ai1 j1 . . . Ai1 jp . . . . . . . . . Aip j1 . . . Aip jp
  • 292.
  • 293.
  • 294.
  • 295.
  • 296.
  • 297.
  • 298.
  • 300.
  • 301.
  • 302.
  • 303.
  • 304.
  • 305.
  • 306.
  • 307. Aip+1,jp+1 . . . Aip+1,jn . . . . . . . . . Ain,jp+1 . . . Ainjn
  • 308.
  • 309.
  • 310.
  • 311.
  • 312.
  • 313.
  • 314.
  • 315. Định lý 1.12 (Chebotarev). Cho p là một số nguyên tố và e = exp(2πi/p). Khi đó tất cả các định thức con của định thức Vandermonde (aij) p−1 i,i=0 là khác không, ở đó aij = eij. Định lý 1.13 (Công thức khai triển Laplace). Cố định p hàng i1, i2, . . . , ip của A với i1 i2 . . . ip. Khi đó det A = ∑ j1...jp,jp+1...jn,ip+1...in (−1)i+j A i1 . . . ip j1 . . . jp ! .A ip+1 . . . in jp+1 . . . jn ! , ở đó i = i1 + . . . + ip, j = j1 + . . . + jp. Đại lượng (−1)i+j A ip+1 . . . in jp+1 . . . jn ! được gọi là phần bù đại số của định thức con A i1 . . . ip j1 . . . jp ! . 2.2 Bài tập Bài tập 1.28. Cho A là một ma trận vuông cấp n. Chứng minh rằng |A + λI| = λn + n ∑ k=1 Skλn−k , ở đó Sk là tổng của tất cả Cn k các định thức con chính cấp k của A.
  • 316. 2. Định thức con và phần phụ đại số 15 Bài tập 1.29. Chứng minh rằng
  • 317.
  • 318.
  • 319.
  • 320.
  • 321.
  • 322.
  • 323.
  • 324.
  • 325.
  • 326. a11 . . . a1n x1 . . . . . . . . . vdots an1 . . . ann xn y1 . . . yn 0
  • 327.
  • 328.
  • 329.
  • 330.
  • 331.
  • 332.
  • 333.
  • 334.
  • 335.
  • 336. = − ∑ i,j xiyj Aij, ở đó Aij là phần bù đại số của phần tử aij. Bài tập 1.30. Chứng minh rằng tổng của các định thức con chính cấp k của AT A bằng tổng bình phương các định thức con chính cấp k của A. Bài tập 1.31. Cho A, B là các ma trận vuông cấp n. Tính    I A C 0 I B 0 0 I    −1 Bài tập 1.32. Tìm một ví dụ một ma trận vuông cấp n mà các phần bù đại số của nó đều bằng 0, ngoại trừ phần tử nằm ở hàng i và cột j. Bài tập 1.33. Cho x và y là các cột có độ dài n. Chứng minh rằng adj(I − xy)T = xyT + (1 − yT x)I. Bài tập 1.34. Cho A là một ma trận phản xứng. Chứng minh rằng adj(A) là một ma trận phản xứng nếu n lẻ và đối xứng nếu n chẵn. Bài tập 1.35. Cho A là một ma trận phản xứng cấp n với các phần tử trên đường chéo chính bằng 1. Tính adj A. Bài tập 1.36. Tìm tất cả các ma trận A có các phần tử không âm sao cho tất cả các phần tử của ma trận A−1 cũng không âm. Bài tập 1.37. Cho e = exp(2πi/n) và A = (aij)n 1 với aij = eij. Tính A−1. Bài tập 1.38. Tìm ma trận nghịch đảo của ma trận Vandermonde V.
  • 337. 16 Chương 1. Ma trận - Định thức §3. PHẦN BÙ SCHUR 3.1 Các định nghĩa và tính chất Cho P = A B C D ! là một ma trận khối, ở đó A, D là các ma trận vuông. Để tính định thức của ma trận P, ta có thể phân tích P dưới dạng sau: A B C D ! = A 0 C I ! I Y 0 X ! = A AY C CY + X ! (1.2) Nếu A là một ma trận khả nghịch thì các phương trình B = AY và D = CY + X có nghiệm lần lượt là Y = A−1B và X = D − CA−1B. Định nghĩa 1.14. Ma trận D − CA−1B được gọi là phần bù Schur của ma trận khả nghịch A trong P, và được kí hiệu là (P|A). Dễ dàng nhận thấy rằng det P = det A det(P|A). Mặt khác, A AY C CY + X ! = A 0 C X ! I Y 0 I ! , (1.3) nên phương trình (??) có thể được viết dưới dạng sau: P = A 0 C P|A ! I A−1B 0 I ! = I 0 CA−1 I ! A 0 0 P|A ! I A−1B 0 I ! (1.4) Tương tự, nếu D là ma trận khả nghịch, thì P = I BD−1 0 I ! A − BD−1C 0 0 D ! I 0 D−1C I ! (1.5) Định lý 1.15. 1. Nếu |A| 6= 0 thì |P| = |A|.|D − CA−1B|; 2. Nếu |D| 6= 0 thì |P| = |A − BD−1C|.|D|. 3. P−1 = A−1 + A−1BX−1CA−1 −A−1BX−1 −X−1CA−1 X−1 ! , ở đó X = (P|A).
  • 338. 3. Phần bù Schur 17 Định lý 1.16. Nếu A và D là các ma trận vuông cấp n, det A 6= 0, và AC = CA, thì |P| = |AD − CB|. Chú ý 1.17. Định lý ?? vẫn đúng nếu |A| = 0, thật vậy, xét ma trận Ae = A + eI. Dễ dàng thấy rằng ma trận Ae sẽ khả nghịch với mọi e đủ nhỏ! Hơn nữa nếu AC = CA thì AeC = CAe. Định lý 1.18. Giả sử u là một hàng, v là một cột, và a là một số bất kì. Khi đó
  • 339.
  • 340.
  • 341.
  • 342.
  • 344.
  • 345.
  • 346.
  • 347.
  • 348. = a|A| − u(adj A)v. Định lý 1.19 (Emily Haynsworth). Cho A =    A11 A12 A13 A21 A22 A23 A31 A32 A33    , B = A11 A12 A21 A22 ! , C = (A11) là các ma trận vuông, và B, C là các ma trận khả nghịch. Khi đó: (A|B) = ((A|C)|(B|C)). 3.2 Bài tập Bài tập 1.39. Cho u và v là các hàng có độ dài n và A là một ma trận vuông cấp n. Chứng minh rằng |A + uT v| = |A| + v(adj A)uT Bài tập 1.40. Cho A là một ma trận vuông. Chứn minh rằng
  • 349.
  • 350.
  • 351.
  • 352.
  • 354.
  • 355.
  • 356.
  • 357.
  • 358. = 1 − ∑ M2 1 + ∑ M2 2 − ∑ M2 3 + . . . ở đó ∑ M2 k là tổng bình phương của tất cả các định thức con cấp k của A.
  • 359. 18 Chương 1. Ma trận - Định thức
  • 360. CHƯƠNG 2 KHÔNG GIAN VÉCTƠ - ÁNH XẠ TUYẾN TÍNH §1. KHÔNG GIAN ĐỐI NGẪU - PHẦN BÙ TRỰC GIAO 1.1 Không gian đối ngẫu Định nghĩa 2.1. Với mỗi không gian véctơ V trên trường K, không gian tuyến tính V∗ mà các phần tử của nó là các ánh xạ tuyến tính trên V, nghĩa là, ánh xạ f : V → K sao cho f (λ1v1 + λ2v2) = λ1 f (v1) + λ2 f (v2) với mọi λ1, λ2 ∈ K và v1, v2 ∈ V, được gọi là không gian đối ngẫu với không gian V. Định lý 2.2. Cho V là không gian Ơclit n chiều, chứng minh điều kiện cần và đủ để ánh xạ f : V → R tuyến tính là tồn tại véctơ a cố định của V để f (x) = a, x , ∀x ∈ V Chú ý 2.3. Khi đó không gian V∗ có thể được coi như đồng nhất với không gian V. Chứng minh. ⇐ Điều kiện đủ: Dễ dàng chứng minh ánh xạ f (x) = a, x , ∀x ∈ V là ánh xạ tuyến tính với mỗi vectơ a cố định đã được chọn trước. ⇒ Điều kiện cần: Giả sử f : V → V là một ánh xạ tuyến tính bất kỳ. (a) Nếu f ≡ 0 thì ta chọn vectơ a = 0 thoả mãn yêu cầu bài toán. (b) Nếu f 6≡ 0. Ta sẽ chứng minh dimKerf = n − 1. Thật vậy, vì f 6≡ 0 nên tồn tại ít nhất một vectơ y ∈ V, y 6∈ Kerf. Cố định một vectơ y như vậy, khi đó với mỗi x ∈ V, đặt λ = f (x) f (y) , z = x − λy = x − f (x) f (y) y thì f (z) = 0 ⇒ z ∈ Kerf . Ta có x = z + λy , tức là mỗi vectơ x ∈ V thừa nhận phân tích thành tổng của 2 vectơ, một vectơ thuộc Kerf và một vectơ thuộc spany. Điều đó có nghĩa là 19
  • 361. 20 Chương 2. Không gian véctơ - Ánh xạ tuyến tính V = Kerf + spany và suy ra dimKerf = n − 1. Bây giờ giả sử V có phân tích thành tổng trực giao V = Kerf + (Kerf )⊥ thì dim (Kerf )⊥ = 1 , tức (Kerf )⊥ = span (y0) , trong đó ky0k = 1 . Đặt a = f (y0) y0 ∈ (Kerf )⊥ , ta sẽ chứng minh vectơ a thoã mãn yêu cầu bài ra, tức là f (x) = a, x , ∀x ∈ V. Thật vậy: Với mỗi x ∈ V, do V = Kerf + (Kerf )⊥ = Kerf + span (y0) nên x = λy0 + y, y ∈ Kerf . Khi đó: f (x) = λf (y0) + f (y) = λf (y0) = λ f (y0) y0, y0 = λ a, y0 = a, λy0 = a, λy0 + y a, y = 0 do a ∈ (Kerf )⊥ , y ∈ (Kerf ) = a, x Với mỗi cơ sở e1, e2, . . . , en của không gian V, đặt e∗ i (ej) = δij, khi đó e∗ 1, . . . , e∗ n sẽ là cơ sở của V∗. Mỗi phần tử f ∈ V∗ khi đó có thể được biểu diễn dưới dạng f = ∑ f (ei)e∗ i . Do đó, nếu cố định cơ sở e1, e2, . . . , en của không gian V, chúng ta có thể xây dựng được một đẳng cấu g : V → V∗ bằng cách đặt g(ei) = e∗ i . Với mỗi ánh xạ tuyến tính A : V1 → V2 toán tử đối ngẫu A∗ : V∗ 2 → V∗ 1 xác định bởi (A ∗ f2)(v1) = f2(Av1) với mỗi f2 ∈ V∗ 2 và v1 ∈ V1. Để thuận tiện hơn, ta kí hiệu f (v) bởi f, v , khi đó định nghĩa của toán tử A∗ có thể được viết lại như sau: hA∗ f2, v1i = hf2, Av1i Gọi (aij)n 1 là ma trận của ánh xạ tuyến tính A trong cặp cơ sở {eα} của V1 và {eβ} của V2, ở đó Aej = ∑i aijei. Tương tự, gọi (a∗ ij)n 1 là ma trận của ánh xạ tuyến tính A∗ trong cặp cơ sở {e∗ β} của V∗ 2 và {e∗ α} của V∗ 1 . Bổ đề 2.4. (a∗ ij) = (aij)T Giả sử {eα} và {eβ} là hai cơ sở khác nhau của không gian véctơ V, A là ma trận chuyển từ cơ sở {eα} sang {eβ}, B là ma trận chuyển cơ sở từ {e∗ α} sang cơ sở {e∗ β}. Bổ đề 2.5. ABT = I. Hệ quả 2.6. Các cơ sở {eα} và {eβ} cảm sinh cùng một đẳng cấu V → V∗ khi và chỉ khi A là một ma trận trực giao, nghĩa là AAT = I.
  • 362. 1. Không gian đối ngẫu - Phần bù trực giao 21 1.2 Phần bù trực giao Định nghĩa 2.7. Với mỗi không gian con W ⊂ V, không gian W⊥ = {f ∈ V∗ | f, w = 0 với mọi w ∈ W}, được gọi là phần bù trực giao của không gian con W. W⊥ là một không gian véctơ con của V∗ và dimW + dimW⊥ = dimV, bởi vì nếu e1, . . . , en là một cơ sở của V sao cho e1, . . . , ek là một cơ sở của W thì khi đó e∗ k+1, . . . , e∗ n là một cơ sở của W⊥. Bổ đề 2.8. 1. Nếu W1 ⊂ W2 thì W⊥ 2 ⊂ W⊥ 1 , 2. (W⊥)⊥ = W, 3. (W1 + W2)⊥ = W⊥ 1 ∩ W⊥ 2 và (W1 ∩ W2)⊥ = W⊥ 1 + W⊥ 2 , 4. Nếu V = W1 ⊕ W2 thì V∗ = W⊥ 1 ⊕ W⊥ 2 . Định lý 2.9. Nếu A : V → V là một toán tử tuyến tính và AW ⊂ W thì A∗W⊥ ⊂ W⊥. Trong không gian các ma trận cỡ m × n, tích trong (tích vô hướng) giữa hai ma trận X, Y được xác định như sau: tr(XYT ) = ∑ i,j xijyij. Định lý 2.10. Cho A là một ma trận cỡ m × n. Nếu với mỗi ma trận X cỡ n × m ta có tr(AX) = 0 thì A = 0. 1.3 Bài tập Bài tập 2.1. Cho ma trận A vuông cấp n thỏa mãn tính chất tr(AX) = 0 với mọi ma trận X có vết bằng 0. Chứng minh rằng A = λI. Bài tập 2.2. Cho A và B là các ma trận cỡ m × n và k × n tương ứng, sao cho nếu AX = 0 thì BX = 0 với X là một véctơ cột nào đó. Chứng minh rằng B = CA, ở đó C là một ma trận cỡ k × m.
  • 363. 22 Chương 2. Không gian véctơ - Ánh xạ tuyến tính §2. HẠT NHÂN VÀ ẢNH - KHÔNG GIAN THƯƠNG 2.1 Hạt nhân và ảnh Định nghĩa 2.11. Cho A : V → W là một ánh xạ tuyến tính từ không gian véctơ V tới không gian véctơ W. Khi đó Ker(A) := {x|x ∈ V, A(x) = 0} được gọi là hạt nhân của A. Im(A) := {y|y ∈ W, ∃x ∈ V, A(x) = y} = {A(x)|x ∈ V} được gọi là ảnh của A. Định lý 2.12. Cho A : V → W là một ánh xạ tuyến tính. Khi đó 1. Ker(A) là một không gian véctơ con của V. 2. Im(A) là một không gian véctơ con của W. 3. dimKerA + dim Im A = dimV. 4. Nếu B = {e1, e2, . . . , en} là một cơ sở của V thì Im(A) = span{A(e1), A(e2), . . . , A(en)}. Cho U A −→ V B −→ W là các ánh xạ tuyến tính. Khi đó Định lý 2.13. dim(Im A ∩ KerB) = dim Im A − dim Im BA = dimKerBA − dimKerA. Định lý 2.14. KerA∗ = (Im A)⊥ và Im A∗ = (KerA)⊥ . Hệ quả 2.15. rank A = rank A∗. Chú ý 2.16. . Nếu V là một không gian Euclide thì V∗ có thể đồng nhất với V, và V = Im A ⊕ (Im A)⊥ = Im A ⊕ KerA∗ = =A∗ ⊕ KerA.
  • 364. 2. Hạt nhân và ảnh - Không gian thương 23 Định lý 2.17 (Fredholm). Cho A : V → V là một toán tử tuyến tính. Xét bốn phương trình sau: (1) Ax = y với x, y ∈ V, (3) Ax = 0, (2) A∗ f = g với f, g ∈ V∗ , (4) A∗ f = 0. Khi đó 1. hoặc là các phương trình (1) và (2) có nghiệm với mọi vế phải, và trong trường hợp này nghiệm là duy nhất 2. hoặc là các phương trình (3) và (4) có cùng số các nghiệm độc lập tuyến tính x1, . . . , xk và f1, . . . , fk và trong trường hợp này các phương trình (1) (và (2) tương ứng) có nghiệm nếu và chỉ nếu f1(y) = . . . = fk(y) = 0 (tương ứng g(x1) = . . . = g(xk) = 0). Định lý 2.18. Phương trình (2) có nghiệm khi và chỉ khi một trong các điều kiện tương đương sau được thỏa mãn a) Tồn tại các ma trận Y và Z sao cho C = AY và C = ZB; b) rank A = rank(A, C) và rank B = rank B C ! Định lý 2.19. Cho rank A = a. Khi đó tồn tại các ma trận khả nghịc L và R sao cho LAR = Ia, ở đó Ia là ma trận cấp n có a phần tử trên đường chéo bằng 1, và các phần tử còn lại bằng 0. 2.2 Không gian thương Nếu W là một không gian véctơ con của không gian V thì V có thể được phân thành lớp các tập con như sau: Mv = {x ∈ V|x − v ∈ W}. Nhận xét rằng Mv = Mv0 nếu và chỉ nếu v − v0 ∈ W. Khi đó trên tập thương V/W = {Mv|v ∈ V}, ta có thể xây dựng một cấu trúc tuyến tính bằng cách đặt λMv = Mλv và Mv + Mv0 = Mv+v0 . Chú ý rằng Mλv và Mv+v0 không phụ thuộc vào cách chọn v và v0 . Định nghĩa 2.20. Không gian V/W được gọi là không gian thương của V modulo W
  • 365. 24 Chương 2. Không gian véctơ - Ánh xạ tuyến tính Để thuận tiện người ta thường kí hiệu lớp Mv bởi v + W. Ánh xạ p : V → V/W, p(v) = Mv, được gọi là phép chiếu chính tắc từ V lên V/W. Hiển nhiên, Kerp = W và Im p = V/W. Bổ đề 2.21. dim(V/W) = dimV − dimW Chứng minh. Nếu e1, . . . , ek là một cơ sở của W và e1, . . . , ek, ek+1, . . . , en là một cơ sở của V thì p(e1) = . . . = p(ek) = 0 và p(ek+1), . . . , p(en) là một cơ sở của V/W. Định lý 2.22. 1. (U/W)/(V/W) ∼ = U/V nếu W ⊂ V ⊂ U; 2. V/V ∩ W ∼ = (V + W)/W nếu V, W ⊂ U. 2.3 Bài tập Bài tập 2.3. Cho A là một toán tử tuyến tính. Chứng minh rằng dimKerAn+1 = dimKerA + n ∑ k=1 dim(Im Ak ∩ KerA) và dim Im A = dim Im An+1 + n ∑ k=1 dim(Im Ak ∩ KerA).
  • 366. 3. Cơ sở của không gian véctơ - Độc lập tuyến tính 25 §3. CƠ SỞ CỦA KHÔNG GIAN VÉCTƠ - ĐỘC LẬP TUYẾN TÍNH 3.1 Bài toán đổi cơ sở Cho A là ma trận của ánh xạ tuyến tính f : V → W trong cặp cơ sở e = {e1, . . . , en} của V và e = {e1, . . . , en} của W. Giả sử e0 = eP và e0 = eQ là các cơ sở khác của V và W. Khi đó A0 = Q−1 AP là ma trận của f trong cặp cơ sở e0 và e0. Đặc biệt, nếu V = W và P = Q thì A0 = P−1AP. Định lý 2.23. Với mỗi toán tử tuyến tính A, đa thức đặc trưng |λI − A| = λn + an−1λn−1 + . . . + a0 không phụ thuộc vào cách chọn cơ sở của không gian V. Định lý 2.24 (Green, 1973). . Cho x1, . . . , xn và y1, . . . , yn là hai cơ sở của không gian V, 1 ≤ k ≤ n. Khi đó k véctơ của y1, . . . , yn có thể hoán đổi được với các véctơ x1, . . . , xk sao cho chúng ta vẫn thu được 2 cơ sở khác của không gian V. Định lý 2.25 (Aupetit, 1988). Cho T là một toán tử tuyến tính trên không gian V sao cho với mọi ξ ∈ V các véctơ ξ, Tξ, . . . , Tnξ là độc lập tuyến tính. Khi đó các toán tử tuyến tính I, T, . . . , Tn là độc lập tuyến tính. 3.2 Bài tập Bài tập 2.4. Trong Vn cho các véctơ e1, . . . , em. Chứng minh rằng nếu m ≥ n + 2 thì tồn tại các số α1, . . . , αm không đồng thời bằng 0 sao cho ∑ αiei = 0 và ∑ αi = 0. Bài tập 2.5. Một tổ hợp tuyến tính lồi của các véctơ v1, . . . , vm là một véctơ bất kì x = t1v1 + . . . + tmvm, ở đó ti ≥ 0 và ∑ ti = 1. Chứng minh rằng trong một không gian véctơ thực n chiều, bất kì tổ hợp tuyến tính lồi của m véctơ cũng là một tổ hợp tuyến tính lồi của không nhiều hơn n + 1 véctơ cho trước. Bài tập 2.6. Chứng minh rằng nếu |aii| ∑k6=i |aik| với mọi i = 1, . . . , n thì A = (aij)n 1 là một ma trận khả nghịch.
  • 367. 26 Chương 2. Không gian véctơ - Ánh xạ tuyến tính Bài tập 2.7. a) Cho các véctơ e1, . . . , en+1 trong một không gian Euclide n chiều, sao cho (ei, ej) 0 nếu i 6= j, chứng minh rằng mọi n véctơ rút ra từ hệ n + 1 véctơ trên đều là một cơ sở của V. b) Chứng minh rằng nếu e1, . . . , em là các véctơ trong Rn sao cho (ei, ej) 0 với i 6= j thì m ≤ n + 1.
  • 368. 4. Hạng của ma trận 27 §4. HẠNG CỦA MA TRẬN Định nghĩa 2.26. Cho A là một ma trận cỡ m × n. Hạng của ma trận A là cấp cao nhất của các định thức con khác không của nó. 4.1 Các tính chất của hạng của ma trận Bổ đề 2.27. 1. rank(A) ≤ min{m, n}. Nếu A là ma trận vuông cấp n thì A khả nghịch khi và chỉ khi rank(A) = n. 2. rank(A + B) ≤ rank(A) + rank(B). Định lý 2.28. (Định lý Sylvester) Cho A, B là các ma trận cỡ m × n và n × p tương ứng. Khi đó rank(AB) = rank(B) − dim(Im B ∩ KerA) Nói riêng, rank(A) + rank(B) − n ≤ rank(AB) ≤ min{rank(A), rank(B)} Chứng minh. Chú ý rằng nếu A là toán tử tuyến tính trên không gian véctơ n chiều thì n = dim Im A + dimKerA và nếu A là ma trận của toán tử tuyến tính A thì rank(A) = dim Im A. Bây giờ xem A như là ánh xạ tuyến tính trên không gian véctơ con Im B. Khi đó ảnh của nó là Im AB và hạt nhân của nó là Im B ∩ KerA. Do đó ta có dim Im AB + dim(Im B ∩ KerA) = dim Im B. Cuối cùng, chú ý rằng bất đẳng thức cần chứng minh được suy ra trực tiếp từ đẳng thức trên và bất đẳng thức sau: dim Im A + dimKer(Im B ∩ KerA) ≤ dim Im A + dimKerA = n. Định lý 2.29 (Bất đẳng thức Frobenius). rank(AB) + rank(BC) ≤ rank(B) + rank(ABC)
  • 369. 28 Chương 2. Không gian véctơ - Ánh xạ tuyến tính Bổ đề 2.30. Hạng của ma trận A bằng cỡ nhỏ nhất của các ma trận B, C, sao cho A = BC. Chứng minh. Nếu A = BC và cỡ nhỏ nhất của các ma trận B, C bằng k. Khi đó ta cần chứng minh rank(A) ≤ min{rank B, rank C} ≤ k. Định lý 2.31 (Flanders, 1962). Cho r ≤ m ≤ n và Mn,m là không gian các ma trận cỡ n × m. Giả sử U là không gian véctơ con của Mn,m thỏa mãn hạng lớn nhất của các phần tử của U bằng r. Khi đó dimU ≤ nr. Bổ đề 2.32. Nếu B ∈ U thì B = B11 B12 B21 0 ! , ở đó B21B12 = 0. Bổ đề 2.33. Nếu B, C ∈ U thì B21C12 + C21B12 = 0. 4.2 Bài tập Bài tập 2.8. Cho A là một ma trận vuông cấp n. CMR tồn tại k ≤ n sao cho r(Ak) = r(Am) với mọi m ≥ k. Nói riêng r(An) = r(An+1). Bài tập 2.9. Giả sử các giá trị riêng của ma trận A có phần thực âm. Chứng minh rằng tồn tại ma trận xác định dương C sao cho ATC + CA = −I. Bài tập 2.10. Cho aij = xi + yj. Chứng minh rằng rank(aij)n 1 ≤ 2. Bài tập 2.11. Cho A là một ma trận vuông có hạng bằng 1. Chứng minh rằng |A + I| = 1 + tr A. Bài tập 2.12. Chứng minh rằng rank(A∗ A) = rank A. Bài tập 2.13. Cho A là một ma trận khả nghịch. Chứng minh rằng nếu rank A B C D ! = rank A thì D = CA−1B. Bài tập 2.14. Cho A1, A2 là các ma trận có cùng cỡ, và V1, V2 là không gian véctơ sinh bởi các hàng của A1, A2 tương ứng, W1, W2 là không gian véctơ sinh bởi các cột của A1, A2 tương ứng. Chứng minh rằng các điều kiện sau là tương đương. 1. rank(A1 + A2) = rank A1 + rank A2,
  • 370. 4. Hạng của ma trận 29 2. V1 ∩ V2 = 0, 3. W1 ∩ W2 = 0. Bài tập 2.15. Chứng minh rằng nếu A và B là các ma trận có cùng cỡ và BT A = 0 thì rank(A + B) = rank A + rank B. Bài tập 2.16. Cho A và B là các ma trận vuông cấp lẻ. Chứng minh rằng nếu AB = 0 thì ít nhất một trong hai ma trận A + AT và B + BT là suy biến.
  • 371. 30 Chương 2. Không gian véctơ - Ánh xạ tuyến tính
  • 372. CHƯƠNG 3 DẠNG CHÍNH TẮC CỦA MA TRẬN VÀ TOÁN TỬ TUYẾN TÍNH §1. VẾT CỦA MA TRẬN Định nghĩa 3.1. Vết của ma trận A là tổng của các phần tử trên đường chéo của nó, được kí hiệu là tr A. Dễ thấy tr AB = ∑ i,j aijbji = tr BA. Do đó, tr PAP−1 = tr P−1 AP = tr A, nghĩa là, hạng của ma trận của toán tử tuyến tính không phụ thuộc vào cách chọn cơ sở của không gian. Nếu A là toán tử tuyến tính trên không gian Euclidean thì: Định lý 3.2. Cho e1, . . . , en là một cơ sở chuẩn tắc. Khi đó tr A = n ∑ i=1 (Aei, ei). 31
  • 373. 32 Chương 3. Dạng chính tắc của ma trận và toán tử tuyến tính §2. CẤU TRÚC CỦA TỰ ĐỒNG CẤU 2.1 Trị riêng và véctơ riêng Giả sử V là một không gian véctơ, f : V → V là một tự đồng cấu của V. Việc nghiên cứu f trên toàn không gian V đôi khi gặp khó khăn, vì V quá lớn. Người ta muốn tránh điều đó bằng cách hạn chế f lên một số lớp không gian con nào đó U của V. Định nghĩa 3.3. Không gian véctơ con U của V được gọi là một không gian con ổn định đối với f (hay một không gian con f ổn định nếu f (U) ⊂ U. Đôi khi người ta nói U là một không gian con ổn định (nếu f đã rõ). Đối với tự đồng cấu f : V → V, các không gian con sau đây là ổn định: {0}, V, Kerf, Im f. Nếu có các không gian con ổn định U1, U2 sao cho V = U1 ⊕ U2 thì f |U1 và f |U2 đều là các tự đồng cấu. Khi đó việc nghiên cứu f trên V có thể quy về việc nghiên cứu fi trên Ui. Nói rõ hơn, nếu f1 có ma trận là A trong cơ sở {e1, e2, . . . , em} của U1 và f2 có ma trận là B trong cơ sở {em+1, . . . , en} của U2 thì f có ma trận A 0 0 B # trong cơ sở {e1, e2, . . . , en} của V. Sau đây ta xét một trường hợp riêng đặc biệt thú vị, có nhiều ứng dụng trong Cơ học, Vật lý. Đó là trường hợp các không gian con ổn định một chiều. Định nghĩa 3.4. Giả sử f là một tự đồng cấu của không gian véctơ V. Nếu có véctơ α 6= 0 và vô hướng λ ∈ R sao cho f (α) = λα, thì λ được gọi là một giá trị riêng của f còn α được gọi là một véctơ riêng ứng với trị riêng λ. Định nghĩa 3.5. Giả sử λ là một trị riêng của tự đồng cấu f : V → V. Không gian véctơ Ker(f − λ idV) gồm tất cả các véctơ riêng ứng với trị riêng λ và véctơ không được gọi là không gian con riêng của f ứng với trị riêng λ. Định nghĩa 3.6. Đa thức bậc n một ẩn X với hệ số thực Pf (X) = det(f − X idV) được gọi là đa thức đặc trưng của tự đồng cấu f. Đa thức bậc n một ẩn X với hệ số thực PA(X) = det(A − X idV) được gọi là đa thức đặc trưng của ma trận A. Nghiệm của đa thức này được gọi là giá trị riêng của A.
  • 374. 2. Cấu trúc của tự đồng cấu 33 Mệnh đề 3.7. Giả sử U là một không gian con ổn định với tự đồng cấu f : V → V. Gọi f : V/U → V/U, f [α] = [f (α)] là đồng cấu cảm sinh bởi f. Khi đó, đa thức đặc trưng của f bằng tích các đa thức đặc trưng của f |U và f . Định lý 3.8. Cho A, B là các ma trận vuông cấp n. Khi đó đa thức đặc trưng của AB và BA là trùng nhau. Hệ quả 3.9. Cho A, B là các ma trận cỡ m × n. Khi đó các đa thức đặc trưng của ABT và BT A khác nhau bởi nhân tử λn−m. Định lý 3.10. Cho ma trận vuông A có tổng của các phần tử trên mỗi cột của nó bằng 1, và giả sử (x1, . . . , xn)T là véctơ riêng của A thỏa mãn x1 + . . . xn 6= 0. Khi đó giá trị riêng tương ứng với véctơ riêng này bằng 1. Định lý 3.11. Nếu tổng của các giá trị tuyệt đối của các phần tử trên mỗi cột của ma trận A không vượt quá 1, thì tất cả các giá trị riêng của nó cũng không vượt quá 1. 2.2 Tự đồng cấu chéo hoá được Định nghĩa 3.12. Tự đồng cấu f của không gian véctơ V được gọi là chéo hoá được nếu có một cơ sở của V gồm toàn các véctơ riêng của f. Nói cách khác f chéo hoá được nếu ma trận của nó trong một cơ sở nào đó của V là một ma trận chéo. Gọi A là ma trận của f trong một cơ sở nào đó của V. Từ định nghĩa ta suy ra f chéo hoá được khi và chỉ khi tồn tại ma trận C khả nghịch sao cho C−1AC là một ma trận chéo. Định nghĩa 3.13. Ma trận A đồng dạng với một ma trận chéo được gọi là ma trận chéo hoá được. Định lý 3.14. Giả sử tự đồng cấu f : V → V có tính chất f2 = f. Khi đó f chéo hoá được. Định lý 3.15 (Điều kiện cần và đủ cho sự chéo hoá). Tự đồng cấu f của không gian véctơ n chiều V chéo hoá được nếu và chỉ nếu hai điều kiện sau đây được thoả mãn: (i) Đa thức đặc trưng của f có đủ nghiệm trong R: Pf (X) = (−1)n (X − λ1)s1 (X − λ2)s2 . . . (X − λm)sm (ii) rank(f − λi idV) = n − si (với i = 1, 2, . . . , m), ở đây si là bội của λi xem như nghiệm của đa thức đặc trưng.
  • 375. 34 Chương 3. Dạng chính tắc của ma trận và toán tử tuyến tính Chú ý 3.16. Trên trường số phức C hầu hết các toán tử tuyến tính đều chéo hóa được, chỉ có các toán tử có trị riêng bội là có thể không chéo hóa được, và các toán tử như vậy sẽ tạo thành một tập hợp có độ đo 0. Nói riêng, tất cả các toán tử unita và Hermitian đều chéo hóa được, và tồn tại một cơ sở trực giao bao gồm các véctơ riêng của nó. Điều này là do nếu AW ⊂ W thì AW⊥ ⊂ W⊥. Chú ý thêm rằng các véctơ riêng của toán tử unita có độ dài bằng 1, vì |Ax| = |x|. Các trị riêng của toán tử Hermitian là thực vì (Ax, x) = (x, Ax) = (Ax, x). Định lý 3.17. Cho A là một ma trận trực giao, khi đó tồn tại một cơ sở trực giao sao cho ma trận của A trong cơ sở đó có dạng đường chéo khối với các khối ±1, hoặc cos ϕ − sin ϕ sin ϕ cos ϕ ! . 2.3 Đa thức tối tiểu Định nghĩa 3.18. Đa thức có bậc nhỏ nhất triệt tiêu ma trận A và có hệ số cao nhất bằng 1 được gọi là đa thức tối tiểu của A. Chú ý 3.19. Đa thức tối tiểu của toán tử tuyến tính được định nghĩa bằng đa thức tối tiểu của ma trận của nó trong một cơ sở nào đó, và không phụ thuộc vào cách chọn cơ sở của không gian. Định lý 3.20. Mọi đa thức triệt tiêu A đều chia hết cho đa thức tối tiểu của nó. Định nghĩa 3.21. Đa thức p được gọi là triệt tiêu véctơ v ∈ V ứng với toán tử tuyến tính A : V → V nếu p(A)v = 0. Đa thức triệt tiêu véctơ v có bậc nhỏ nhất và có hệ số cao nhất bằng 1 được gọi là đa thức tối tiểu của v. Định lý 3.22. Với mọi toán tử tuyến tính A : V → V, tồn tại một véctơ v ∈ V sao cho đa thức tối tiểu của nó ứng với A trùng với đa thức tối tiểu của toán tử A. Định lý 3.23 (Cayley - Hamilton). Mỗi ma trận đều là nghiệm của đa thức đặc trưng của nó. Định lý 3.24 (Farahat, Lederman, 1958). Đa thức đặc trưng của ma trận A cấp n trùng với đa thức tối tiểu của nó khi và chỉ khi với mọi véctơ (x1, . . . , xn) tồn tại các cột P và Q có độ dài n sao cho xk = QT AkP. Định lý 3.25 (Greenberg, 1984). Cho pA(t) là đa thức đặc trưng của ma trận A, và X là ma trận giao hoán với A. Khi đó pA(X) = M(A − X), ở đó M là một ma trận giao hoán với A và X.
  • 376. 2. Cấu trúc của tự đồng cấu 35 2.4 Bài tập Bài tập 3.1. Chứng minh rằng các hệ số của đa thức đặc trưng của ma trận A có thể được mô tả như sau: det(A − λI) = (−λ)n + c1(−λ)n−1 + . . . + cn trong đó ck là tổng của tất cả các định thức con chính cấp k của ma trận A. (Một định thức con được gọi là chính nếu các chỉ số hàng và chỉ số cột của nó trùng nhau). Bài tập 3.2. Giả sử λ là nghiệm bội p của đa thức đặc trưng của ma trận A vuông cấp n. a. Chứng minh rằng số chiều của không gian riêng ứng với giá trị riêng λ không lớn hơn p. b. Đặt r = rank(A − λI). Chứng minh rằng 1 ≤ n − r ≤ p. Bài tập 3.3. Chứng minh rằng các giá trị riêng của ma trận A−1 bằng bằng nghịch đảo của các giá trị riêng của A (kể cả bội). Chứng minh. det(A−1 − λI) = (−λ)n . det(A−1 ). det A − 1 λ I . Bài tập 3.4. Chứng minh rằng các giá trị riêng của ma trận A2 bằng bằng bình phương của các giá trị riêng của A (kể cả bội). Chứng minh. Giả sử λ1, λ2, . . . , λn là các giá trị riêng của ma trận A (kể cả bội). Khi đó det(A − λI) = (λ1 − λ)(λ2 − λ) . . . (λn − λ) det(A + λI) = (λ1 + λ)(λ2 + λ) . . . (λn + λ) Nhân hai vế của các đẳng thức trên với nhau ta có det(A2 − λ2 I) = (λ2 1 − λ2 )(λ2 2 − λ2 ) . . . (λ2 n − λ2 ) thay λ2 bởi λ trong đẳng thức trên ta có det(A2 − λI) = (λ2 1 − λ)(λ2 2 − λ) . . . (λ2 n − λ) suy ra điều phải chứng minh. Bài tập 3.5. Chứng minh rằng các giá trị riêng của ma trận Ap bằng luỹ thừa p của các giá trị riêng của A (kể cả bội).
  • 377. 36 Chương 3. Dạng chính tắc của ma trận và toán tử tuyến tính Chứng minh. Tương tự như bài tập trên, gọi 1, e1, e2, . . . , en−1 là các căn bậc p khác nhau của 1 ek = cos 2kπ p + i sin 2kπ p . Ta có det(A − λI) = (λ1 − λ)(λ2 − λ) . . . (λn − λ) det(A − λe1 I) = (λ1 − λe1)(λ2 − λe1) . . . (λn − λe1) . . . det(A − λep−1 I) = (λ1 − λep−1)(λ2 − λep−1) . . . (λn − λep−1) Nhân các vế của p phương trình trên với nhau ta được det(An − λp I) = (λ p 1 − λp )(λ p 2 − λp ) . . . (λ p n − λp ) Thay λp bởi λ trong đẳng thức trên ta suy ra điều phải chứng minh. Bài tập 3.6. Cho A là một ma trận vuông cấp n với các phần tử trong một trường đóng đại số K. Gọi λ1, λ2, . . . , λn là các giá trị riêng (kể cả bội) của ma trận A. Chứng minh rằng nếu f (X) là một đa thức với các hệ số trong K thì det f (A) = f (λ1)f (λ2) . . . f (λn). Chứng minh. Theo bài ra, det(A − λI) = n ∏ i=1 (λi − λ). Giả sử f (λ) = a0 s ∏ j=1 (λ − µi), khi đó f (A) = a0 s ∏ j=1 (A − µi I). Vậy det f (A) = an 0 s ∏ j=1 det(A − µj I) = an 0 s ∏ j=1 n ∏ i=1 (λi − muj) = n ∏ i=1 a0 s ∏ j=1 (λi − µj) # = n ∏ i=1 f (λi) Bài tập 3.7. Chứng minh rằng nếu PA(λ) = k ∏ i=1 (λi − λ)si là đa thức đặc trưng của A thì đa thức đặc trưng của f (A) với f là một đa thức là Pf (A)(λ) = k ∏ i=1 (f (λi − λ))si .
  • 378. 2. Cấu trúc của tự đồng cấu 37 Chứng minh. Áp dụng bài tập ?? với hàm số g(x) = f (x) − λ, giả sử PA(λ) = det(A − λI) = n ∏ i=1 (λi − λ). (kể cả bội), ta có det g(A) = n ∏ i=1 g(λi) hay Pf (A)(λ) = det(f (A) − λI) = n ∏ i=1 (f (λi) − λ) Bài tập 3.8. Chứng minh rằng nếu λ1, λ2, . . . , λn là các giá trị riêng của ma trận A và f (X) là một đa thức thì f (λ1), f (λ2), . . . , f (λn) là các trị riêng của ma trận f (A). Chứng minh. Là một hệ quả trực tiếp của bài tập ??. Bài tập 3.9. Chứng minh rằng nếu λ1, λ2, . . . , λn là các giá trị riêng của ma trận A và f (x) = g(x) h(x) là một phân thức hữu tỷ xác định khi x = A (nghĩa là det h(A) 6= 0), khi đó det f (A) = f (λ1)f (λ2) . . . f (λn) và f (λ1), f (λ2), . . . , f (λn) là các giá trị riêng của ma trận f (A). Chứng minh. Áp dụng đẳng thức det f (A) = det(g(A) det(h(A) và sử dụng các kết quả của bài tập ??, ??, ??. Bài tập 3.10. Chứng minh rằng nếu A, B là các ma trận vuông cùng cấp thì các đa thức đặc trưng của AB và BA trùng nhau. Chứng minh. i) Nếu A là ma trận không suy biến thì AB và BA là các ma trận đồng dạng nên có cùng đa thức đặc trưng. ii) Nếu A suy biến thì 0 là một giá trị riêng của A. Chọn m đủ lớn sao cho Ak = A − 1 k I không suy biến với mọi k ≥ m. (kể từ m đủ lớn nào đó, 1 k , k ≥ m không phải là giá trị riêng của A). Khi đó AkB và BAk có cùng đa thức đặc trưng. Cho k → ∞ ta có điều phải chứng minh. Bài tập 3.11. Cho P là một ma trận hoán vị. Chứng minh rằng Pn = I; PT = P−1. Tìm các giá trị riêng của P. Bài tập 3.12. a) Có tồn tại hay không các ma trận vuông cùng cấp A và B sao cho AB − BA = I?
  • 379. 38 Chương 3. Dạng chính tắc của ma trận và toán tử tuyến tính b) Chứng minh rằng nếu AB − BA = A thì |A| = 0. Bài tập 3.13. Tìm trị riêng và véctơ riêng của ma trận A = (aij) với aij = λi/λj. Bài tập 3.14. Chứng minh rằng mọi ma trận vuông A đều là tổng của hai ma trận khả nghịch. Bài tập 3.15. Chứng minh rằng các trị riêng của ma trận phụ thuộc liên tục vào các phần tử của nó. Nói rõ hơn, cho trước ma trận A = (aij). Khi đó với mọi e 0, tồn tại δ 0 sao cho nếu |aij − bij| δ và λ là một trị riêng của A, thì tồn tại một trị riêng µ của B = (bij) sao cho |λ − µ| e. Bài tập 3.16. Tổng của các phần tử trên mỗi hàng của ma trận khả nghịch A bằng s. Chứng minh rằng tổng của các phần tử trên mỗi hàng của A−1 bằng 1/s. Bài tập 3.17. Chứng minh rằng nếu hàng đầu tiên của ma trận S−1AS có dạng (λ, 0, 0, . . . , 0), thì cột đầu tiên của ma trận S là một véctơ riêng của ma trận A ứng với véctơ riêng λ. Bài tập 3.18. Cho f (λ) = |λI − A|, ở đó A là một ma trận vuông cấp n. Chứng minh rằng f 0(λ) = n ∑ i=1 |λi − Ai|, ở đó Ai là ma trận thu được từ A bằng cách bỏ đi hàng thứ i và cột thứ j. Bài tập 3.19. Cho λ1, . . . , λn là các trị riêng của ma trận A. Chứng minh rằng các trị riêng của ma trận adj A là Πi6=1, . . . , Πi6=n. Bài tập 3.20. Véctơ x được gọi là đối xứng (phản đối xứng, tương ứng) nếu các tọa độ của nó thỏa mãn (xi = xn−i) (tương ứng (xi = −xn−i)). Cho A = (aij) là một ma trận đối xứng tâm (aij = an−j,n−j). Chứng minh rằng trong các véctơ riêng của A ứng với các trị riêng, tồn tại hoặc là một véctơ riêng đối xứng, hoặc là một véctơ riêng phản đối xứng. Bài tập 3.21. Cho ma trận A vuông, phức cấp n có các phần tử ai,n−i+1 = xi có thể khác 0, còn các phần tử còn lại bằng 0. Tìm điều kiện của {x1, . . . , xn} sao cho ma trận A chéo hóa được. Bài tập 3.22 (Drazin, Haynsworth, 1962). a) Chứng minh rằng ma trận A có m véctơ riêng độc lập tuyến tính tương ứng với các trị riêng thực khi và chỉ khi tồn tại ma trận vuông S xác định không âm có hạng bằng m sao cho AS = SA∗. b) Chứng minh rằng ma trận A có m véctơ riêng độc lập tuyến tính tương ứng với các trị riêng λ thỏa mãn |λ| = 1 khi và chỉ khi tồn tại ma trận vuông S xác định không âm có hạng bằng m sao cho ASA∗ = S.
  • 380. 2. Cấu trúc của tự đồng cấu 39 Bài tập 3.23. Cho A là một ma trận cấp n và f1(A) = A − (tr A)I, fk+1(A) = fk(A)A − 1 k + 1 tr(fk(A)A)I. Chứng minh rằng fn(A) = 0. Bài tập 3.24. Cho A và B là các ma trận cấp n. Chứng minh rằng nếu tr Am = tr Bm với mọi m = 1, . . . , n thì các trị riêng của A và B là trùng nhau. Bài tập 3.25. Cho A là một ma trận khả nghịch và đa thức đặc trưng của nó trùng với đa thức tối tiểu. Chứng minh rằng đa thức tối tiểu của A−1 bằng p(0)−1λn p(λ−1). Bài tập 3.26. Cho Π(x − λi)ni là đa thức tối tiểu của ma trận A. Chứng minh rằng đa thức tối tiểu của ma trận A I 0 A ! là Π(x − λi)ni+1.
  • 381. 40 Chương 3. Dạng chính tắc của ma trận và toán tử tuyến tính §3. DẠNG CHUẨN CỦA MA TRẬN 3.1 Dạng chuẩn Jordan của ma trận Định lý 3.26. Cho A và B là các ma trận vuông thực và A = P−1BP, ở đó P là một ma trận phức. Khi đó tồn tại ma trận thực Q sao cho A = Q−1BQ. Điều đó có nghĩa là tập hợp tất cả các ma trận có dạng A = P−1BP với P là một ma trận phức thì ”không lớn hơn” tập hợp tất cả các ma trận có dạng A = Q−1BQ với Q là một ma trận thực. Một khối Jordan cỡ r × r là một ma trận có dạng Jr(λ) =            λ 1 0 . . . . . . 0 0 λ 1 . . . . . . 0 . . . . . . ... ... ... . . . 0 0 0 . . . 1 0 0 0 0 . . . λ 1 0 0 0 . . . 0 λ            Định nghĩa 3.27. Một ma trận Jordan là một ma trận khối có các khối Jordan trên đường chéo. Một cơ sở Jordan của toán tử tuyến tính A : V → V là một cơ sở của không gian V sao cho ma trận của nó trong cơ sở đó là một ma trận Jordan. Định lý 3.28 (Jordan). Với mọi toán tử tuyến tính A : V → V trên C, tồn tại một cơ sở Jordan, và ma trận Jordan của nó được xác định duy nhất, sai khác hoán vị các khối Jordan của nó. Chú ý 3.29. Dạng chuẩn Jordan của một ma trận được sử dụng một cách rất thuận tiện trong việc thực hiện phép tính lũy thừa một ma trận. Cụ thể hơn, nếu A = P−1 JP thì An = P−1 JnP. Để tính lũy thừa của khối Jordan Jr(λ) = λI + N, ta có công thức khai triển Newton: Jn = n ∑ k=0 Cn k λk Nn−k , Chú ý 3.30. Cơ sở Jordan luôn luôn tồn tại trên các trường đóng đại số. Trên R không phải lúc nào cũng tồn tại cơ sở Jordan. Tuy nhiên trên trường số thực cũng tồn tại một dạng Jordan, là thực hóa dạng chuẩn Jordan trên trường số phức.
  • 382. 3. Dạng chuẩn của ma trận 41 Định lý 3.31. Với mỗi toán tử tuyến tính A trên trường số thực, luôn tồn tại một cơ sở mà ma trận của nó trong cơ sở đã cho có dạng đường chéo khối với các khối Jm1 (t1), . . . , Jmk (tk) tương ứng với trị riêng thực ti và các khối J∗ n1 (λ1), . . . , J∗ ns (λs) tương ứng với trị riêng phức λi và λ̄i, ở đó J∗ n(λ) là ma trận cỡ 2n × 2n thu được từ khối Jordan Jn(λ) bằng cách thay thế mỗi phần tử có dạng a + ib của nó bởi ma trận a b −b a ! . Chú ý 3.32. Từ dạng chuẩn Jordan, mỗi toán tử tuyến tính A trên C đều có thể được phân tích dưới dạng A = As + An, ở đó As là chéo hóa được, và An là luỹ linh, hơn nữa AsAn = An As. Định lý 3.33. Các toán tử As và An là xác định duy nhất, hơn nữa As = S(A) và An = N(A), ở đó S và N là các đa thức nào đó. Định lý 3.34. Cho A là một toán tử tuyến tính khả nghịch trên trường số phức C. Khi đó A có thể biểu diễn dưới dạng A = AsAu = Au As, ở đó As là toán tử chéo hóa được và Au là một toán tử lũy đơn (toán tử lũy đơn là tổng của toán tử đồng nhất và toán tử lũy linh). Biểu diễn này là duy nhất. Chứng minh. Nếu A khả nghịch thì As cũng khả nghịch. Khi đó A = As + An = As Au, ở đó Au = A−1 s (As + An) = I + A−1 s An. 3.2 Dạng chuẩn Frobenius Dạng chuẩn Jordan chỉ là một trong số nhiều dạng chuẩn khác nhau của ma trận của ánh xạ tuyến tính. Một trong những dạng chuẩn khác được giới thiệu trong mục này là dạng tuần hoàn, hay còn gọi là dạng chuẩn Frobenius. Một khối Frobenius hay khối tuần hoàn là một ma trận có dạng         0 0 0 . . . 0 −a0 1 0 0 . . . 0 −a1 0 1 0 . . . 0 −a2 . . . . . . ... ... ... . . . 0 0 0 . . . 1 −an−1         (3.1) Nếu A : Vn → Vn và Ae1 = e2, . . . , Aen−1 = en thì ma trận của A đối với cơ sở e1, . . . , en là một khối Frobenius. Định lý 3.35. Với mọi toán tử tuyến tính A : V → V trên trường số thực hoặc phức, tồn tại một cơ sở của V mà ở đó ma trận của A trong cơ sở đó là ma trận có dạng đường chéo khối với các khối Frobenius.
  • 383. 42 Chương 3. Dạng chính tắc của ma trận và toán tử tuyến tính Bổ đề 3.36. Đa thức đặc trưng của khối Frobenius (??) trùng với đa thức tối tiểu của nó, và bằng λn + n−1 ∑ k=0 akλk. 3.3 Bài tập Bài tập 3.27. Chứng minh rằng A và AT là các ma trận đồng dạng. Bài tập 3.28. Cho σ(i), i = 1, . . . , n là một phép hoán vị bất kì và P = (pij), ở đó pij = δiσ(j). Chứng minh rằng ma trận P−1AP thu được từ ma trận A bằng cách phép hoán vị σ các cột và các hàng của nó. Ma trận P được gọi là ma trận hoán vị tương ứng với σ. Bài tập 3.29. Cho ma trận A có m trị riêng phân biệt, m 1. Đặt bij tr(Ai+j). Chứng minh rằng |bij|m−1 0 6= 0 và |bij|m 0 = 0 Bài tập 3.30. Chứng minh rằng rank A = rank A2 nếu và chỉ nếu tồn tại lim λ→0 (A + λI)−1 A. Bài tập 3.31. Cho A là một khối Jordan. Chứng minh rằng tồn tại một véctơ v sao cho các véctơ v, Av, A2v, . . . , An−1v tạo nên một cơ sở của không gian V. Khi đó ma trận của A trong cơ sở đó là ma trận có dạng khối Frobenius. Bài tập 3.32. Với mỗi khối Frobenius A tồn tại một ma trận đối xứng S sao cho A = SATS−1.
  • 384. 4. Biểu diễn ma trận 43 §4. BIỂU DIỄN MA TRẬN 4.1 Rút gọn một ma trận về ma trận dạng đường chéo đơn giản Nhận xét rằng phép biến đổi A 7→ XAX−1 bảo toàn vết của ma trận A. Chính vì vậy mà các phần tử trên đường chéo của ma trận XAX−1 không thể tùy ý. Định lý sau đây cho phép chúng ta rút gọn đường chéo của ma trận A 6= λI về dạng đơn giản (0, 0, . . . , 0, tr A). Định lý 3.37 (Gibson, 1975). Cho A 6= λI. Khi đó ma trận A đồng dạng với ma trận đường chéo diag(0, 0, . . . , 0, tr A). Định lý 3.38. Cho A là một ma trận phức bất kì. Khi đó tồn tại ma trận unita U sao cho các phần tử trên đường chéo của ma trận UAU−1 là bằng nhau. Định lý 3.39 (Marcus, Purves, 1959). Mọi ma trận vuông A khác 0 đều đồng dạng với một ma trận mà tất cả các phần tử trên đường chéo của nó đều khác 0. 4.2 Biểu ma trận dưới dạng tọa độ cực Cũng giống như mỗi một số phức z = a + bi đều có thể được biểu diễn dưới dạng z = |z|.eiϕ, mỗi ma trận S đều có thể được biểu diễn dưới dạng A = SU, trong đó S là một ma trận Hermitian và U là một ma trận unita. Định lý 3.40. Mọi ma trận vuông A trên R (hoặc C) đều có thể biểu diễn được dưới dạng A = SU, ở đó S là một ma trận symmetric (Hermitian) xác định không âm, và U là một ma trận trực giao (unita). Nếu A khả nghịch thì phân tích trên là duy nhất. Chú ý 3.41. Tương tự, chúng ta cũng có thể xây dựng được một phân tích A = U1S1 ở đó S1 là ma trận symmetric (Hermitian) và U1 là ma trận trực giao (unita). Khi đó S = S1 khi và chỉ khi AA∗ = A∗A, nghĩa là A là một ma trận chuẩn tắc. Định lý 3.42. Mọi ma trận vuông A đều có thể biểu diễn được dưới dạng A = UDW, ở đó U và W là các ma trận unita và D là một ma trận đường chéo. Định lý 3.43. Cho A = S1U1 = U2S2 là các biểu diễn của ma trận khả nghịch A dưới dạng tọa độ cực. Khi đó U1 = U2.
  • 385. 44 Chương 3. Dạng chính tắc của ma trận và toán tử tuyến tính 4.3 Biểu diễn Schur Định lý 3.44 (Schur). Mọi ma trận vuông A trên trường số phức C đều có thể được biểu diễn dưới dạng A = UTU∗, ở đó U là ma trận unita và T là ma trận tam giác, hơn nữa, A là chuẩn tắc nếu và chỉ nếu T là ma trận đường chéo. 4.4 Biểu diễn Lanczos Định lý 3.45 (Lanczos, 1958). Mọi ma trận thực cỡ m × n có hạng p 0 đều có thể được biểu diễn dưới dạng A = XΛYT ở đó X và Y là các ma trận cỡ m × p và n × p với các cột trực giao và Λ là ma trận đường chéo cỡ p × p. 4.5 Bài tập Bài tập 3.33. Chứng minh rằng với mọi ma trận vuông A khác 0 đều tồn tại ma trận X sao cho các ma trận X và A + X có các trị riêng khác 0 đều khác nhau. Bài tập 3.34. Chứng minh rằng mọi phép biến đổi tuyến tính trên Rn là tổ hợp của một phép biến đổi trực giao và một phép co giãn dọc theo hướng vuông góc (với các hệ số phân biệt). Bài tập 3.35. Cho A : Rn → Rn là một toán tử co, nghĩa là |Ax| ≤ |x|. Coi Rn là không gian con của R2n. Chứng minh rằng A là hạn chế lên Rn của tổ hợp của phép một phép biến đổi trực giao trên R2n và một phép chiếu lên Rn. Bài tập 3.36 (Phân tích Gauss). Giả sử mọi định thức con |aij| p 1, p = 1, . . . , n của ma trận A vuông cấp n bằng 0. Chứng minh rằng A có thể biểu diễn dưới dạng A = T1T2, ở đó T1 là ma trận tam giác dưới và T2 là một ma trận tam giác trên. Bài tập 3.37 (Phân tích Gram). Chứng minh rằng mọi ma trận khả nghịch X có thể biểu diễn được dưới dạng X = UT, ở đó U là một ma trận trực giao và T là một ma trận tam giác trên. Bài tập 3.38 (Ramakrishnan, 1972). Cho B = diag(1, e, . . . , en−1), ở đó e = exp(2πi n , và C = (cij)n 1, ở đó cij = δi,j−1 (ở đây j − 1 được xem như là modulo n). Chứng minh rằng mọi ma trận M vuông cấp n trên C được biểu diễn duy nhất dưới dạng M = ∑n−1 k,l=0 akl BkCl. Bài tập 3.39. Chứng minh rằng mọi ma trận phản xứng A có thể được biểu diễn dưới dạng A = S1S2 − S2S1, ở đó S1 và S2 là các ma trận đối xứng.
  • 386. CHƯƠNG 4 CÁC MA TRẬN CÓ DẠNG ĐẶC BIỆT §1. MA TRẬN ĐỐI XỨNG - MA TRẬN HERMITIAN 1.1 Các định nghĩa và tính chất Định nghĩa 4.1. Ma trận thực A được gọi là đối xứng nếu A = AT. Ma trận phức A được gọi là Hermitian nếu A∗ = A, ở đó A∗ = ĀT. Các tính chất đơn giản: 1. Các giá trị riêng của ma trận Hermitian đều thực. 2. Mọi ma trận Hermitian A đều có thể được phân tích dưới dạng U∗DU, ở đó U là ma trận unita và D là ma trận đường chéo. 3. Ma trận A là ma trận Hermitian khi và chỉ khi (Ax, x) ∈ R với mọi véctơ x. Mỗi ma trận đối xứng A có một dạng toàn phương q(x) = xT Ax và dạng song tuyến tính B(x, y) = xT Ay tương ứng. Định nghĩa 4.2. Trong trường hợp thực, nếu A là một ma trận đối xứng, thì dạng toàn phương xT Ax được gọi là xác định dương nếu xT Ax 0 với mọi x 6= 0. Ma trận A khi đó được gọi là ma trận xác định dương, và viết A 0. Trong trường hợp phức, nếu A là một ma trận Hermitian, thì dạng Hermitian (hay còn gọi là dạng nửa song tuyến tính) x∗Ax được gọi là xác định dương nếu x∗Ax 0 với mọi x 6= 0. Ma trận A khi đó được gọi là ma trận xác định dương, và viết A 0. 45
  • 387. 46 Chương 4. Các ma trận có dạng đặc biệt Chú ý 4.3. Nếu A là một ma trận Hermitian, giả sử U là ma trận thỏa mãn A = U∗DU, ở đó D là một ma trận đường chéo. Khi đó x∗Ax = (Ux)∗D(Ux). Khi đó bằng phép đổi biến y = Ux, dạng Hermitian có thể biểu diễn dưới dạng sau: ∑λiyiyi = ∑λi|yi|2 . Khi đó nếu A xác định dương thì vết của nó, λ1 + . . . + λn và định thức của nó λ1 . . . λn đều dương. Định lý 4.4 (Tiêu chuẩn Sylvester). Cho A = (aij) là một ma trận Hermitian. Khi đó A xác định dương khi và chỉ khi tất cả các định thức con chính của nó đều dương. Định lý 4.5. Nếu A là một ma trận xác định không âm, và x là một véctơ sao cho x∗Ax = 0. Khi đó Ax = 0. Định lý 4.6. Cho A là một ma trận xác định dương, B là một ma trận Hermitian. Khi đó AB là một ma trận chéo hóa được, và số các trị riêng dương, âm và bằng 0 của nó bằng với số các giá trị tương ứng của B. Định lý 4.7. Mọi ma trận chéo hóa được với các trị riêng thực đều có thể biểu diễn được dưới dạng tích của một ma trận xác định dương và một ma trận Hermitian. 1.2 Bài tập Bài tập 4.1. Chứng minh rằng mọi ma trận Hermitian có hạng bằng r đều có thể biểu diễn được dưới dạng tổng của r ma trận Hermitian có hạng bằng 1. Bài tập 4.2. Chứng minh rằng nếu A là một ma trận xác định dương, thì adj A cũng là một ma trận xác định dương. Bài tập 4.3. Chứng minh rằng nếu A là một ma trận Hermitian khác 0 thì rank A ≥ (tr A)2 tr(A2) . Bài tập 4.4. Cho A là một ma trận xác định dương cấp n. Chứng minh rằng ∞ Z −∞ e−xT Ax dx = ( √ π)n |A|−1/2 , Bài tập 4.5. Chứng minh rằng nếu hạng của một ma trận đối xứng (Hermitian) bằng r thì nó có r định thức con chính khác 0. Bài tập 4.6. Cho S là một ma trận đối xứng khả nghịch cấp n có tất cả các phần tử đều dương. Hỏi phần tử lớn nhất khác 0 có thể của ma trận S−1 bằng bao nhiêu?
  • 388. 2. Ma trận phản xứng 47 §2. MA TRẬN PHẢN XỨNG 2.1 Các định nghĩa và tính chất Định nghĩa 4.8. Ma trận thực A được gọi là phản xứng nếu AT = −A. Chú ý rằng ma trận phản xứng cấp n lẻ có định thức bằng 0. Định lý 4.9. Nếu A là một ma trận phản xứng thì A2 là một ma trận đối xứng, xác định không dương. Hệ quả 4.10. Các trị riêng khác 0 của ma trận phản xứng là thuần ảo. Định lý 4.11. Điều kiện xT Ax = 0 thỏa mãn với mọi x khi vào chỉ khi A là một ma trận phản xứng. Bổ đề 4.12. Hạng của một ma trận phản xứng là một số chẵn. Định nghĩa 4.13. Toán tử tuyến tính A trên không gian Euclidean được gọi là phản xứng nếu ma trận của nó trong một cơ sở trực chuẩn nào đó là phản xứng. Định lý 4.14. Đặt Λi = 0 −λi λi 0 ! . Với mọi toán tử tuyến tính phản xứng A, tồn tại một cơ sở trực chuẩn sao cho ma trận của nó trong cơ sở trực chuẩn đó có dạng diag(Λ1, Λ2, . . . , Λk, 0, . . . , 0). 2.2 Bài tập Bài tập 4.7. Chứng minh rằng nếu A là một ma trận phản xứng thì I + A là một ma trận khả nghịch. Bài tập 4.8. Cho A là một ma trận phản xứng, khả nghịch. Chứng minh rằng A−1 cũng là một ma trận phản xứng. Bài tập 4.9. Chứng minh rằng mọi nghiệm của đa thức đặc trưng của ma trận AB, ở đó A và B là các ma trận phản xứng cấp 2n, đều có bội lớn hơn 1.
  • 389. 48 Chương 4. Các ma trận có dạng đặc biệt §3. MA TRẬN TRỰC GIAO - PHÉP BIẾN ĐỔI CAYLEY 3.1 Các định nghĩa và tính chất Định nghĩa 4.15. Ma trận thực A được gọi là trực giao, nếu AAT = I. Nhận xét rằng nếu A là một ma trận trực giao, thì các hàng (và các cột) của nó là một hệ trực chuẩn. Hơn nữa, ma trận A trực giao khi và chỉ khi (Ax, Ay) = (x, y) với mọi x, y. Nếu A là một ma trận trực giao, thì nó cũng là một ma trận unita, do đó các trị riêng của nó có giá trị tuyệt đối bằng 1. Các trị riêng của ma trận trực giao nằm trên vòng tròn đơn vị có tâm là gốc tọa độ, các trị riêng của ma trận phản xứng nằm trên trục ảo. Trong giải tích phức, phép biến đổi f (z) = 1−z 1+z biến vòng tròn đơn vị thành trục ảo và f (f (z)) = z, vì thế, chúng ta hy vọng rằng phép biến đổi f (A) = (I − A)(I + A)−1 sẽ biến một ma trận trực giao thành ma trận phản xứng. Phép biến đổi này được gọi là phép biến đổi Cayley. Đặt A# = (I − A)(I + A)−1, dễ dàng thấy rằng (A#)# = A. Định lý 4.16. Phép biến đổi Cayley biến mọi ma trận phản xứng thành ma trận trực giao và biến mọi ma trận trực giao A với |A + I| 6= 0 thành ma trận phản xứng. Định lý 4.17 (Hsu, 1953). Với mọi ma trận vuông A, tồn tại ma trận J = diag(±1, . . . , ±1) sao cho |A + J| 6= 0. 3.2 Bài tập Bài tập 4.10. Chứng minh rằng nếu p(λ) là đa thức đặc trưng của một ma trận trực giao vuông cấp n thì λn p(λ−1 ) = ±p(λ). Bài tập 4.11. Chứng minh rằng mọi ma trận unita cấp 2 với định thức bằng 1 đều có dạng u v −v̄ ū ! , ở đó |u|2 + |v|2 = 1. Bài tập 4.12. Cho A là một ma trận trực giao vuông cấp 3 và có định thức bằng 1. Chứng minh rằng a) (tr A)2 − tr(A)2 = 2 tr A. b) (∑ i aii − 1)2 + ∑ ij (aij − aji)2 = 4.
  • 390. 3. Ma trận trực giao - Phép biến đổi Cayley 49 Bài tập 4.13. Cho J là một ma trận khả nghịch. Ma trận A được gọi là J- trực giao nếu AT JA = J và J-phản xứng nếu AT J = −JA. Chứng minh rằng phép biến đổi Cayley biến một ma trận J- trực giao thành một ma trận J- phản xứng và ngược lại. Bài tập 4.14 (Djokovíc, 1971). Giả sử tất cả các giá trị tuyệt đối của các trị riêng của ma trận A bằng 1 và |Ax| ≤ |x| với mọi x. Chứng minh rằng A là một toán tử unita. Bài tập 4.15 (Zassenhaus, 1961). Một toán tử unita U biến véctơ khác không x thành véctơ Ux trực giao với x. Chứng minh rằng mọi cung của vòng tròn đơn vị chứa tất cả các trị riêng của U có độ dài không nhỏ hơn π.
  • 391. 50 Chương 4. Các ma trận có dạng đặc biệt §4. MA TRẬN CHUẨN TẮC 4.1 Các định nghĩa và tính chất Toán tử tuyến tính A trên C được gọi là chuẩn tắc, nếu A∗A = AA∗. Ma trận của một toán tử chuẩn tắc trong một cơ sở trực chuẩn được gọi là ma trận chuẩn tắc. Hiển nhiên nếu A là ma trận chuẩn tắc thì A∗A = AA∗. Định lý 4.18. Các điều kiện sau là tương đương: 1. A là ma trận chuẩn tắc, 2. A = B + iC, ở đó B, C là các ma trận Hermitian giao hoán, 3. A = UΛU∗, ở đó U là ma trận unita và Λ là ma trận đường chéo. 4. n ∑ i=1 |λ2 i | = n ∑ i,j=1 |a2 ij|, ở đó λ1, . . . , λn là các trị riêng của A. Định lý 4.19. Nếu A là một ma trận chuẩn tắc, thì KerA∗ = KerA và Im A∗ = Im A. Hệ quả 4.20. Nếu A là một ma trận chuẩn tắc, thì V = KerA + ⊕(KerA)⊥ = KerA ⊕ Im A Định lý 4.21. Ma trận A là chuẩn tắc nếu và chỉ nếu mọi véctơ riêng của A cũng là véctơ riêng của A∗. Định lý 4.22. Nếu ma trận A là chuẩn tắc, thì A∗ có thể biểu diễn dưới dạng như là một đa thức của A. Hệ quả 4.23. Nếu A và B là các ma trận chuẩn tắc và AB = BA thì A∗B = BA∗ và AB∗ = B∗ A, nói riêng AB cũng là một ma trận chuẩn tắc. 4.2 Bài tập Bài tập 4.16. Cho A là một ma trận chuẩn tắc. Chứng ming rằng tồn tại ma trận chuẩn tắc B sao cho A = B2. Bài tập 4.17. Cho A và B là các toán tử chuẩn tắc sao cho Im A ⊥ Im B. Chứng minh rằng A + B là một toán tử chuẩn tắc. Bài tập 4.18. Chứng minh rằng ma trận A là chuẩn tắc nếu và chỉ nếu A∗ = AU, ở đó U là ma trận unita.
  • 392. 4. Ma trận chuẩn tắc 51 Bài tập 4.19. Chứng minh rằng nếu A là một toán tử chuẩn tắc và A = SU là biểu diễn trong tọa độ cực của nó thì SU = US. Bài tập 4.20. Cho A, B và AB là các ma trận chuẩn tắc. Chứng minh rằng BA cũng là một ma trận chuẩn tắc.
  • 393. 52 Chương 4. Các ma trận có dạng đặc biệt §5. MA TRẬN LUỸ LINH 5.1 Các định nghĩa và tính chất Định nghĩa 4.24. Ma trận A vuông cấp n được gọi là luỹ linh nếu tồn tại số nguyên k sao cho Ak = 0. Nếu có thêm Ak−1 6= 0 thì k được gọi là bậc lũy linh của ma trận A. Định lý 4.25. Bậc luỹ linh của một ma trận lũy linh bằng cấp cao nhất của các khối Jordan của nó. Định lý 4.26. Cho A là ma trận luỹ linh, vuông cấp n. Khi đó An = 0. Định lý 4.27. Đa thức đặc trưng của một ma trận vuông cấp n lũy linh bằng λn. Định lý 4.28. Cho A là một ma trận vuông cấp n. Chứng minh rằng A lũy linh khi và chỉ khi tr(Ap) = 0 với mọi p = 1, 2, . . . , n. Định lý 4.29. Cho A : V → V là một toán tử tuyến tính và W là một không gian con bất biến của V. Đặt A1 : W → W và A2 : V/W → V/W là các toán tử cảm sinh bởi toán tử A. Chứng minh rằng nếu A1 và A2 là lũy linh thì A cũng là lũy linh. 5.2 Bài tập Bài tập 4.21. A là ma trận lũy linh nếu và chỉ nếu tất cả giá trị riêng của A đều bằng 0. Bài tập 4.22. Chứng minh rằng nếu A là ma trận lũy linh thì I − A là ma trận khả nghịch. Chứng minh. Ta có I = I − Ak = (I − A)(I + A + A2 + . . . + Ak−1 ) Bài tập 4.23. Chứng minh rằng với mọi ma trận vuông A luôn có thể phân tích A = B + C với C là một ma trận lũy linh và B là ma trận chéo hóa được và BC = CB. Bài tập 4.24. Cho A và B là các ma trận vuông cùng cấp thỏa mãn B là ma trận lũy linh và AB = BA. Chứng minh rằng det(A + B) = det A Bài tập 4.25. Cho A và B là các ma trận vuông cùng cấp thỏa mãn A2008 = I; B2009 = 0 và AB + 4A + 2009B = 0. Chứng minh rằng (A + B) là ma trận không suy biến.
  • 394. 5. Ma trận luỹ linh 53 Bài tập 4.26. (2000) Cho A và B là các ma trận vuông cùng cấp thỏa mãn A1999 = 0; B2000 = 0 và AB = BA. Chứng minh rằng (A + B + I) khả nghịch. Chứng minh. Nhận xét rằng (A + B)3999 = 0 nên (A + B) là ma trận luỹ linh, suy ra điều phải chứng minh. Bài tập 4.27. Cho A và B là các ma trận vuông cùng cấp thỏa mãn A1999 = I; B2000 = I và AB = BA. Chứng minh rằng (A + B + I) khả nghịch. Chứng minh. Giả sử (A + B + I) suy biến. Khi đó tồn tại vecto X khác 0 sao cho (A + B + I)X = 0. Hay (A + I)X = −BX suy ra (A + I)1999X = −B1999X = −X, suy ra ((A + I)1999 + I)X = 0. Theo gỉa thiết (A2000 − I)x = 0. Ta sẽ chứng minh hai đa thức (x + 1)1999 + 1 và x2000 − 1 là nguyên tố cùng nhau. Thậy vậy, giả sử chúng có nghiệm chung là z. Khi đó (z + 1)1999 = −1 và z2000 = 1. Từ đó suy ra môđun của z và (z + 1) đều là 1. Do đó, arg z = ±2π 3 và z2000 = cos ±4000π 3 + sin ±4000π 3 = cos ±4π 3 + sin ±4π 3 6= 1 Vậy tồn tại các đa thức P(x) và Q(x) để P(x)[(x + 1)1999 + 1] + Q(x)(x2000 − 1) = 1 Từ đó suy ra [P(A)[(A + 1)1999 + 1] + Q(A)(A2000 − I)]X = X hay X = 0, mâu thuẫn với việc chọn X. Vậy ta có điều phải chứng minh Bài tập 4.28. (IMC) Cho hai ma trận vuông cấp n, A và B. Giả sử tồn tại (n + 1) số t1, t2, . . . , tn phân biệt sao cho các ma trận Ci = A + tiB là các ma trận lỹ linh với mọi i = 1, ..., n + 1. Chứng minh rằng A và B cũng là các ma trận lũy linh Bài tập 4.29. Tìm các ma trận A, B sao cho λA + µB là luỹ linh với mọi λ, µ nhưng không tồn tại ma trận P sao cho P−1AP và P−1BP là các ma trận tam giác.
  • 395. 54 Chương 4. Các ma trận có dạng đặc biệt §6. TOÁN TỬ CHIẾU - MA TRẬN LŨY ĐẲNG 6.1 Các định nghĩa và tính chất Định nghĩa 4.30. Toán tử P được gọi là toán tử chiếu (hay luỹ đẳng) nếu P2 = P. Định lý 4.31. Tồn tại một cơ sở của không gian sao cho ma trận của toán tử chiếu có dạng diag(1, . . . , 1, 0, . . . , 0). Hệ quả 4.32. Có một tương ứng 1-1 giữa toán tử chiếu và phân tích V = W1 ⊕ W2 của không gian V. Nói rõ hơn, với mỗi phân tích V = W1 ⊕ W2, tồn tại toán tử chiếu P thỏa mãn P(w1 + w2) = w1; và ngược lại, với mỗi toán tử chiếu P có một phân tích tương ứng V = Im P ⊕ KerP. Toán tử P khi đó có thể được gọi là toán tử chiếu lên W1 theo hướng W2. Hệ quả 4.33. Nếu P là toán tử chiếu thì rank P = tr P. Hệ quả 4.34. Nếu P là toán tử chiếu thì I − P cũng là một toán tử chiếu, hơn nữa Ker(I − P) = Im P và Im(I − P) = KerP. Định lý 4.35. Toán tử chiếu P là Hermitian nếu và chỉ nếu Im P ⊥ KerP. Định lý 4.36. Toán tử chiếu P là Hermitian nếu và chỉ nếu |Px| ≤ x với mọi x. Các toán tử chiếu Hermitian P và Q được gọi là trực giao nếu Im P ⊥ Im Q, nghĩa là PQ = QP = 0. Định lý 4.37. Cho P1, . . . , Pn là các toán tử chiếu Hermitian. Khi đó toán tử P = P1 + . . . + Pn là toán tử chiếu nếu và chỉ nếu PiPj = 0 với mọi i 6= j. Định lý 4.38 (Djokovíc, 1971). Cho V = V1 ⊕ . . . ⊕ Vk, ở đó Vi 6= 0 với mọi i = 1, . . . , k. Đặt Pi : V → Vi là các phép chiếu trực giao và A = P1 + . . . + Pk. Khi đó 0 ≤ |A| ≤ 1, và |A| = 1 nếu và chỉ nếu Vi ⊥ Vj với mọi i 6= j. 6.2 Bài tập Bài tập 4.30. Cho P là một toán tử chiếu và V = Im P ⊕ KerP. Chứng minh rằng nếu Im P ⊥ KerP thì Pv là hình chiếu trực giao của v lên Im P. Bài tập 4.31. Cho A là một ma trận vuông cấp n. Chứng minh rằng các điều kiện sau là tương đương
  • 396. 6. Toán tử chiếu - Ma trận lũy đẳng 55 a. A là ma trận lũy đẳng. b. Cn = Im A + KerA với Ax = x với mọi x ∈ Im A. c. KerA = Im(I − A) d. rank(A) + rank(I − A) = n e. Im(A) ∩ Im(I − A) = {0} Bài tập 4.32. Cho A là một ma trận vuông cấp n. Chứng minh rằng A là lũy đẳng khi và chỉ khi rank(A) = tr(A) và rank(I − A) = tr(I − A). Bài tập 4.33. Cho P1 và P2 là các toán tử chiếu. Chứng minh rằng 1. P1 + P2 là toán tử chiếu khi và chỉ khi P1P2 = P2P1 = 0. 2. P1 − P2 là toán tử chiếu khi và chỉ khi P1P2 = P2P1 = P2. Bài tập 4.34 (Định lý ergodic). Cho A là ma trận unita. Chứng minh rằng lim n→∞ 1 n n−1 ∑ i=0 Ai x = Px, ở đó P là một phép chiếu Hermitian lên Ker(A − I). Bài tập 4.35. Cho A và B là các ma trận vuông cấp n. Chứng minh rằng nếu AB = A và BA = B thì A, B là các ma trận lũy đẳng. Bài tập 4.36. Cho A và B là các ma trận vuông cấp n, lũy đẳng. Tìm điều kiện cần và đủ để (A + B) là ma trận lũy đẳng. Bài tập 4.37. Cho A là ma trận lũy đẳng. Chứng minh rằng (A + I)k = I + (2k − 1)A với mọi k ∈ N. Bài tập 4.38. (OL) Cho A, B là các ma trận cùng cấp, lũy đẳng và AB + BA = 0. Tính det(A − B). Bài tập 4.39. Cho A, B là các ma trận cùng cấp, lũy đẳng và I − (A + B) khả nghịch. CMR tr(A) = tr(B). Bài tập 4.40. Cho A1, A2, . . . , Ak là các toán tử tuyến tính trên không gian véctơ n chiều V sao cho A1 + A2 + . . . + Ak = I. Chứng minh rằng nếu các điều kiện sau là tương đương 1. A1, . . . , Ak là các toán tử chiếu.
  • 397. 56 Chương 4. Các ma trận có dạng đặc biệt 2. Ai Aj = 0 với mọi i 6= j. 3. rank A1 + . . . + rank Ak = n. Bài tập 4.41. Cho A1, A2, . . . , Ak là các ma trận lũy đẳng. Chứng minh rằng nếu A1 + A2 + . . . + Ak = I thì Ai Aj = 0 với mọi i 6= j.
  • 398. 7. Ma trận đối hợp 57 §7. MA TRẬN ĐỐI HỢP Định nghĩa 4.39. Toán tử tuyến tính (hoặc ma trận) A được gọi là đối hợp nếu A2 = I. Dễ dàng kiểm chứng rằng P là ma trận lũy đẳng nếu và chỉ nếu 2P − I là ma trận đối hợp. Định lý 4.40. Tồn tại một cơ sở của không gian sao cho ma trận của toán tử đối hợp có dạng diag(±1, . . . , ±1). Chú ý 4.41. Nếu A là toán tử đối hợp thì V = Ker(A + I) ⊕ Ker(A − I). Định lý 4.42 (Djokovíc, 1967). Ma trận A có thể biểu diễn được dưới dạng tích của 2 ma trận đối hợp nếu và chỉ nếu các ma trận A và A−1 là đồng dạng. Hệ quả 4.43. Nếu B là một ma trận khả nghịch sao cho XT BX = B thì X có thể biểu diễn được dưới dạng tích của 2 ma trận đối hợp. Nói riêng, mọi ma trận trực giao đều có thể biểu diễn được dưới dạng tích của 2 ma trận đối hợp. Bài tập 4.42. Chứng minh rằng A là ma trận đối hợp nếu và chỉ nếu 1 2(I + A) là ma trận lũy đẳng.
  • 399. 58 Chương 4. Các ma trận có dạng đặc biệt §8. MA TRẬN HOÁN VỊ (HAY CÒN GỌI LÀ MA TRẬN GIAO HOÁN) 8.1 Định nghĩa Định nghĩa 4.44. Ma trận hoán vị là ma trận có dạng C =         c0 c1 c2 . . . cn−1 cn−1 c0 c1 . . . cn−2 cn−2 cn−1 c0 . . . cn−3 . . . . . . . . . ... . . . c1 c2 c3 . . . c0         Ma trận P =          0 1 0 . . . 0 0 0 1 . . . 0 . . . . . . . . . ... . . . 0 0 0 ... 1 1 0 0 . . . 0          được gọi là ma trận hoán vị cơ sở. 8.2 Bài tập Bài tập 4.43. Chứng minh rằng Pn = I; PT = P−1. Tìm các giá trị riêng của P. Bài tập 4.44. Cho f (x) = c0 + c1x + ... + cn−1xn−1. Chứng minh rằng a. C = f (P) b. Các giá trị riêng của C là f (ωk), k = 0, 1, ..., n − 1. với ω là căn bậc n của 1. c. det C = n−1 ∏ i=0 f (ωi) Bài tập 4.45. Cho A, B là các ma trận hoán vị. Chứng minh rằng A và B giao hoán và AB cũng là một ma trận hoán vị. Bài tập 4.46. Cho A là một ma trận hoán vị. Chứng minh rằng rank(Ak) = rank(A) với mọi k.
  • 400. CHƯƠNG 5 CÁC BẤT ĐẲNG THỨC MA TRẬN §1. CÁC BẤT ĐẲNG THỨC CHO MA TRẬN ĐỐI XỨNG VÀ HERMITIAN 1.1 Các định lý cơ bản Định nghĩa 5.1. Cho A, B là các ma trận Hermitian. Ta viết A B (tương ứng A ≥ B) nếu A − B là ma trận xác định dương (tương ứng xác định không âm). Định lý 5.2. Nếu A B 0 thì A−1 B−1. Định lý 5.3. Nếu A 0 thì A + A−1 ≥ 2I. Định lý 5.4. Nếu A là ma trận thực và A 0 thì (A−1 x, x) = max y (2(x, y) − (Ay, y)). Định lý 5.5. Cho A = A1 B B∗ A2 ! 0. Khi đó det A ≤ det A1 det A2. Hệ quả 5.6 (Bất đẳng thức Hadamard). Nếu A = (aij) là ma trận xác định dương, thì det A ≤ a11a22 . . . ann và dấu bằng xảy ra khi và chỉ khi A là ma trận đường chéo. Hệ quả 5.7. Nếu X là một ma trận bất kì, thì | det X| ≤ ∑ i |x1i|2 . . . |xni|2 . 59
  • 401. 60 Chương 5. Các bất đẳng thức ma trận Định lý 5.8. Cho A = A1 B B∗ A2 ! 0 là ma trận xác định dương, ở đó B là một ma trận vuông. Khi đó | det B|2 ≤ det A1 det A2. Định lý 5.9. Cho αi 0, ∑ αi = 1 và Ai 0. Khi đó |α1A1 + . . . + αk Ak| ≥ |A1|α1 . . . |Ak|αk . Định lý 5.10. Cho λi là các số phức bất kì và Ai ≥ 0. Khi đó | det(λ1 A1 + . . . + λk Ak)| ≤ det(|λ1|A1 + . . . + |λk|Ak). Định lý 5.11. Cho A và B là các ma trận thực xác định dương, và A1, B1 là các ma trận thu được từ ma trận A, B tương ứng bằng cách xóa đi các hàng đầu tiên và cột đầu tiên của nó. Khi đó |A + B| |A1 + B1| ≥ |A| |A1| + |B| |B1| 1.2 Bài tập Bài tập 5.1. Cho A và B là các ma trận vuông cấp n 1, ở đó A 0 và B ≥ 0. Chứng minh rằng |A + B| ≥ |A| + |B| và dấu bằng xảy ra khi và chỉ khi B = 0. Bài tập 5.2. Cho A và B là các ma trận Hermitian và A 0. Chứng minh rằng det A ≤ | det(A + iB)| và dấu bằng xảy ra khi và chỉ khi B = 0. Bài tập 5.3. Cho Ak và Bk là các ma trận con cấp k ở phía trên, góc trái của các ma trận xác định dương A và B sao cho A B. Chứng minh rằng |Ak| |Bk|. Bài tập 5.4. Cho A và B là các ma trận thực đối xứng và A ≥ 0. Chứng minh rằng nếu C = A + iB là ma trận không khả nghịch, thì Cx = 0 với x là một véctơ thực khác 0 nào đó. Bài tập 5.5. Cho A là ma trận vuông cấp n và A 0. Chứng minh rằng |A|1/n = min 1 n tr(AB), ở đó giá trị nhỏ nhất được lấy trên tất cả các ma trận B xác định dương có định thức bằng 1. Bài tập 5.6. Cho A là ma trận thực đối xứng xác định dương. Chứng minh rằng det       0 x1 . . . xn x1 . . . xn A       ≤ 0
  • 402. 2. Các bất đẳng thức cho trị riêng 61 §2. CÁC BẤT ĐẲNG THỨC CHO TRỊ RIÊNG 2.1 Các bất đẳng thức cơ bản Định lý 5.12 (Bất đẳng thức Schur). Cho λ1, . . . , λn là các trị riêng của ma trận A = (aij)n 1. Khi đó n ∑ i=1 |λi|2 ≤ n ∑ i,j=1 |aij|2 , và dấu bằng xảy ra nếu và chỉ nếu A là một ma trận chuẩn tắc. Định lý 5.13. Cho λ1, . . . , λn là các trị riêng của ma trận A = B + iC, ở đó B và C là các ma trận Hermitian. Khi đó n ∑ i=1 | Re λi|2 ≤ n ∑ i,j=1 |bij|2 và n ∑ i=1 | Im λi|2 ≤ n ∑ i,j=1 |cij|2 . Định lý 5.14 (H. Weyl). Cho A và B là các ma trận Hermitian, C = A + B. Cho các trị riêng của các ma trận trên được xếp theo thứ tự tăng dần lần lượt là α1 ≤ . . . ≤ αn, β1 ≤ . . . ≤ βn, γ1 ≤ . . . ≤ γn. Khi đó a) γi ≥ αj + βi−j+1 với i ≥ j, b) γi ≤ αj + βi−j+n với i ≤ j. Định lý 5.15. Cho A = B C C∗ D ! là một ma trận Hermitian. Giả sử các trị riêng của A và B được xếp theo thứ tự tăng dần sau: α1 ≤ . . . ≤ αn, β1 ≤ . . . ≤ βm. Khi đó αi ≤ βj ≤ αi+n−m. Định lý 5.16. Cho A và B là các phép chiếu Hermitian, nghĩa là A2 = A và B2 = B. Khi đó các trị riêng của AB là thực và nằm trong khoảng [0, 1]. Định nghĩa 5.17. Các giá trị σi = √ µi, ở đó µi là các trị riêng của ma trận A∗A, được gọi là các giá trị kì dị của ma trận A. Chú ý 5.18. Nếu A là một ma trận Hermitian xác định không âm thì các giá trị kì dị của A và các trị riêng của A là trùng nhau. Nếu A = SU là phân tích trong tọa độ cực của A, thì các giá trị kì dị của A trùng với các trị riêng của ma trận S. Với ma trận S, tồn tại một ma trận unita V sao cho S = VΛV∗, ở đó Λ là ma trận đường chéo. Do đó, mọi ma trận A có thể được biểu diễn dưới dạng A = VΛW, ở đó V và W là các ma trận unita và Λ = diag(σ1, . . . , σn).